Jump to content

Wikipedia:Reference desk/Humanities

From Wikipedia, the free encyclopedia

This is an old revision of this page, as edited by 71.104.139.75 (talk) at 20:04, 6 December 2008 (→‎Is spying illegal?). The present address (URL) is a permanent link to this revision, which may differ significantly from the current revision.

Welcome to the humanities section
of the Wikipedia reference desk.
Select a section:
Want a faster answer?

Main page: Help searching Wikipedia

   

How can I get my question answered?

  • Select the section of the desk that best fits the general topic of your question (see the navigation column to the right).
  • Post your question to only one section, providing a short header that gives the topic of your question.
  • Type '~~~~' (that is, four tilde characters) at the end – this signs and dates your contribution so we know who wrote what and when.
  • Don't post personal contact information – it will be removed. Any answers will be provided here.
  • Please be as specific as possible, and include all relevant context – the usefulness of answers may depend on the context.
  • Note:
    • We don't answer (and may remove) questions that require medical diagnosis or legal advice.
    • We don't answer requests for opinions, predictions or debate.
    • We don't do your homework for you, though we'll help you past the stuck point.
    • We don't conduct original research or provide a free source of ideas, but we'll help you find information you need.



How do I answer a question?

Main page: Wikipedia:Reference desk/Guidelines

  • The best answers address the question directly, and back up facts with wikilinks and links to sources. Do not edit others' comments and do not give any medical or legal advice.
See also:


November 29

Famous quote

There is a quote out there that goes something like "To men of rationale, the world appears happy. To men of emotions, the world appears sad." Or, it might go something like "The world is a happy place to those with greater rationale, while it is a sad place to those with greater emotion." This is not exactly how it goes, and I cannot remember who said it, but any help would be appreciated. Thanks. —Preceding unsigned comment added by 173.48.62.20 (talk) 00:40, 29 November 2008 (UTC)[reply]

Horace Walpole is credited with "Life is a comedy to those who think, and a tragedy to those who feel" (see [1]). -- JackofOz (talk) 02:02, 29 November 2008 (UTC)[reply]
Thank you! That's exactly what I was looking for!

A shadow detaching from its owner, and turning into an evil being

I remember as a child hearing a radio theatre story about a person whose shadow somehow got detached from its owner. The shadow transformed into an evil being, that turned against its former master. Recently, I read Las luces de septiembre by Carlos Ruíz Zafón, in which this theme is an important part of the plot. Now, Zafón wasn't even born at the time I listened to this, so the theme is certainly older. My question is, in which previous legends/stories/novels has this theme been used, and can someone point to an original version of the story? --NorwegianBlue talk 00:48, 29 November 2008 (UTC)[reply]

The idea turns up in Peter Pan, as I recall, but I have no idea if that's the original occurrence (I don't think Pan's shadow was particularly evil, just mischievous). --Tango (talk) 00:52, 29 November 2008 (UTC)[reply]
Check out Living Shadow at tvtropes.org. The oldest example they have seems to be The Shadow (fairy tale), but it's hard not to believe that it's much much older. -- BenRG (talk) 01:17, 29 November 2008 (UTC)[reply]
In David Eddings Belgariad and Mallorean series, sorcerors could send their shadows away from themselves but they'd stay under their control for the most part. Eddings tends to use the standard story forms from medieval romance, taking various elements, themes and archetypes from them and writing his own stories so he probably borrowed the idea from earlier writings. Exxolon (talk) 01:30, 29 November 2008 (UTC)[reply]
I also recall in the Fighting Fantasy gamebook Midnight Rogue at one point you walk through a magical torch's light and your shadow detaches from you and attacks you. You could fight it or try other methods of dealing with the problem. Exxolon (talk) 01:52, 29 November 2008 (UTC)[reply]
Thanks! After reading the synopsis of The Shadow (fairy tale), I'm pretty sure that's the one I heard. The article mentions the story Peter Schlemihl by Adelbert von Chamisso as a precedent. However, in that story, the main character sells his shadow to the devil, and it is the lack of a shadow that leads to misery; the shadow itself doesn't come back to haunt its former master. Zafón explicitly references the Doppelgänger motif in his novel, although Doppelgängers are generally thought of as separate beings, not detached shadows. Another related motif that comes to mind is vampires' lack of a shadow and reflection. I agree that the detached shadow theme is probably lot older than the 1800's. --NorwegianBlue talk 12:11, 29 November 2008 (UTC)[reply]

In Lord Dunsany's Charwoman's Shadow the detached shadow is not evil as such, but a person who has consented to having his shadow detached can be condemned to hell... AnonMoos (talk) 17:15, 29 November 2008 (UTC)[reply]

Hate poems

I’m looking for poems which express hatred or rage. Suggestions? Thanks, --S.dedalus (talk) 06:06, 29 November 2008 (UTC)[reply]

I guess my preferred angry poetry, such as that of Heinrich Heine, lets hatred and rage sublimate into biting sarcasm. I suspect that is not what you are looking for, however, so how about Marina Tsvetaeva? Choosing an unusual object of lyrical hatred, she sounds quite angry in "Readers of Newspapers". ---Sluzzelin talk 07:20, 29 November 2008 (UTC)[reply]
If it's that good in English, it must be incredible in Russian. Oy. Antandrus (talk) 15:44, 29 November 2008 (UTC)[reply]

death metal lyrics —Preceding unsigned comment added by 82.3.145.61 (talk) 07:34, 29 November 2008 (UTC)[reply]

Robert Browning wrote a famous one to Edward Fitzgerald, that came to mind immediately.[2] "Soliloquy of the Spanish Cloister" [3] is another one by him.

Ogden Nash wrote a humorous one praising hate in general (try searching on "hating, my boy, is an art") but that's probably not what you want.

If folk songs count, "Here's to the state of Richard Nixon" is brutal. 67.122.210.149 (talk) 08:31, 29 November 2008 (UTC)[reply]

Catullus is unsurpassed; I don't think anyone approaches his ferocity. And if it's an ex- you're after, even better. Unexpurgated translations into English did not even appear until mid-20th century. It's strong stuff. I like the Peter Whigham translation (Penguin) pretty much. Antandrus (talk) 13:56, 29 November 2008 (UTC)[reply]
Another would be the opening several hundred lines of François Villon's Grand Testament. He'd just gotten out of prison at Meung where he'd been horribly abused, and wasn't too happy about the bishop who put him there. Antandrus (talk) 14:00, 29 November 2008 (UTC)[reply]
I'd also suggest the lyrics to "Death on two legs" by Queen, which starts "You suck my blood like a leech/Break the law and you breach/Screw my brain till it hurts" and contains the line "You're a sewer rat decaying in a cesspool of pride". I've never seen a more concentrated flow of vitriol in a poem! --TammyMoet (talk) 14:49, 29 November 2008 (UTC)[reply]
Wow -- there's a lot. Depending on the exact focus and scope of what you are looking for:
The Iliad, which is, after all, about rage, and still retains the power to shock after almost three thousand years;
Dante, The Inferno, much of which is saturated with hate and rage eternal;
Paradise Lost, many of the speeches of Satan, particularly that splendid condemnation of the Sun, I think in Book IV, where he tells it how much he hates it for what it represents of Divine Light and what he has lost;
Shakespeare is full of good passages: check out Coriolanus, for example his denunciation of the people as they are banishing him, and Timon of Athens, and then the rages of King Lear. Antandrus (talk) 15:14, 29 November 2008 (UTC)[reply]
(ec) I'll suggest O Fortuna as a general rant against fate: "Fate - monstrous and empty / you whirling wheel / you are malevolent / well-being is vain / and always fades to nothing." or, if you prefer a more poetic, but less accurate translation: "Fate, as vicious as capricious / You're a wheel whirling around: / Evil doings, worthless wooings, / Crumble away to the ground." And so on. Of course, it sounds more impressive sung in Latin by a choir, while backed by an orchestra. Matt Deres (talk) 15:15, 29 November 2008 (UTC)[reply]
Book IV of the Aeneid, when Dido goes nuts and kills herself. Adam Bishop (talk) 15:41, 29 November 2008 (UTC)[reply]

If you are looing for something more current, why not listen to any number of jamaican dance hall artists- hate music at its finest! (although i second the catullus vote -he had some serious sass!).82.22.4.63 (talk) 15:47, 29 November 2008 (UTC)[reply]

Check out the lyrics to Ludo's "Love Me Dead", which includes such lovely lines as "You're a parasitic, psycho, filthy creature finger-bangin' my heart". Little Red Riding Hoodtalk 23:20, 29 November 2008 (UTC)[reply]

Try Howl by Allen Ginsberg. DOR (HK) (talk) 08:31, 1 December 2008 (UTC)[reply]

Otto Nadolski

Dear Editor,

I have recently written a short description of who Otto Nadolski was.

The following thread has been declined, reasoning that the person is not of significance.

http://en.wikipedia.org/wiki/Wikipedia_talk:Articles_for_creation/Submissions/Otto_Nadolski

This is evidently a false judgement, because there is sufficient evidence of the contrary.

1) He is mentioned as one of the most notable professors in wiki page http://en.wikipedia.org/wiki/Lviv_Polytechnic also his is the only name still red, asking for further elaboration.

2) He has been mentioned in other Wiki pages in other languages.

3) Sufficient evidence has been given , as he was mentioned in official literature published by the Lviv Politechnic. (see source on proposed article).

4) He was by far of more importance than the other notable professors mentioned on the page, as they were appointed by Otto Nadolski himself.

5) Being three time Rector of a politechnic described as: "Since its foundation in 1844 it was one of the most important centres of science and technological development in Central Europe" deserves sufficient importance to be mentioned in Wikipedia.

I dont know why this evident information is not sufficient for Wikipedia, I therefore suggest reconsidering the article, taking the above into account.

daniel29680 / [email address removed] —Preceding unsigned comment added by Daniel29680 (talkcontribs) 10:46, 29 November 2008 (UTC)[reply]

This isn't really the place to appeal such decisions. The main requirement for a person to be notable is that they are mentioned is reliable, secondary sources (something from his own university doesn't really qualify). If you can find a newspaper article about him or maybe something saying he's been awarded a major prize in his field, then he may be notable enough for an article. The guideline for academics is here. If, after reading that, you are confident he is notable and you have sources to verify it, then I suggest you just create the article yourself (you already have an account, so you just need to wait about 4 days until you'll be allowed to create articles), or you can make another request at Articles for Creation detailing the sources you've found. --Tango (talk) 13:08, 29 November 2008 (UTC)[reply]

British police operation

Opposition MP Damian Green released too much information embarrassing to the government, so the police was sent to arrest him. The leader of the Opposition, the Speaker, and the mayor of London were told beforehand, but the Prime Minister insists neither he or any other government ministers were informed. [4] Is it usual for the police to brief the lower level politicians but not the higher level ones? 121.72.170.238 (talk) 11:06, 29 November 2008 (UTC)[reply]

Denials, yes, but isn't there a story which is relevant? Angus McLellan (Talk) 13:53, 29 November 2008 (UTC)[reply]
I'm not an expert on UK constitutional law or practice, but I would say what is suggested her is entirely believable. The problem appears to be your assumption 'lower level politicians' and 'higher level ones'. Okay I'm not sure about the Mayor of London bit but informing the leader of the Opposition is understandable since Damian Green belong to his party so it would likely be considered prudent to inform him that one of his MPs is being arrested. (If it were a Liberal-Democrat being arrested, it seems entirely plausible that the Leader of Opposition wouldn't have been informed, instead simply the leader of the LibDems.) The Speaker is responsible for handling all matters concerning the conduct of MPs therefore he or she should generally be informed if a MP is being arrested. The PM however should have little to do with the arrest and while it may be normal to inform him in very high profile cases, e.g. when people are being arrested for terrorism, it may also be considered prudent not to involve him in any matters concerning alleged wrongdoing by opposition MPs to avoid the implication that the police are helping the government. They do have independence in such matters after all. In other words, your mistake is thinking of lower level politicans and higher level politicians when what you should be thinking about is their role and why they were or were not informed. As said, I'm not so sure about why the Mayor of London was informed, but perhaps because this involved the Metropolitan Police carrying out raids inside London. Nil Einne (talk) 13:36, 1 December 2008 (UTC)[reply]
The Mayor of London is responsible for policing in London, as chair of the Metropolitan Police Authority. The closest precedent is the cash for honours inquiry, but it was only Labour politicians and supporters arrested then[5] so it is hard to apply exactly to the current case. --Maltelauridsbrigge (talk) 14:27, 1 December 2008 (UTC)[reply]

National Bankruptcies..

Hello Wikipedia,

I'd just like to understand a little more about what it means for countries to go bankrupt.. I understand that all countries borrow form others and that some countries debts are higher than others. When some countries feel that they won't get their money back, they stop lending so there is a 'cash-flow' problem on a national scale. Assuming thats about right (and if its not, let me know!) how likely is it that the UK will soon face possible bankruptcy? (i've just read an article on the Guardian website which suggests its quite likely - which is obvioulsy terrifying!) Further to that, he suggests that joining the Euro would be the easiest escape route - i'm not sure i understand why... (the article is here [6])..

a quick economics lesson would be much appreciated!

Thanks, 82.22.4.63 (talk) 13:13, 29 November 2008 (UTC)[reply]

I believe the usual term is "defaulting" on their loans, for countries. That simply means they don't pay what they owe, as opposed to "bankruptcy", which is a legal process which makes it no longer necessary to pay all of your debts. Understand that the government of one country doesn't typically borrow money from the governments of others countries, but rather from individuals and corporations and pension funds from all over the world (including their own nation), in the form of government bonds. So, defaulting on those loans harms everyone, not just foreign governments. There may be some benefits to the nation which defaults, such as not having to pay off the interest each year, so being able to apply that money to the regular budget, instead. Also, not being able to borrow money in the future may finally force that nation to live within it's means. I'm not sure if the positives outweigh the negatives, however. StuRat (talk) 15:06, 29 November 2008 (UTC)[reply]
I'm not sure of the proportions, but a significant amount of government debt is held by other governments in the form of foreign reserves (eg. China's government holds large amounts of US government debt). It's not necessarily desirable for a country to "live within its means" (it's a rather controversial topic, there are plenty of people the disagree with what follows) - as long as debt increases no faster than the economy grows, the country remains in a strong financial situation with a higher standard of living than they would have had with lower public spending (or higher taxes, or both). Saving up only makes sense if you expect to need to spend those savings in the future - as long as you keep your debt low enough and your credit worthiness high enough so that you can afford to borrow more when needed, there is little need for reserves. It's all a matter of weighing up the benefits of spending more now with the risks of need to borrow more in the future - borrowing in small amounts can easily be the right decision (borrowing in large amounts should, of course, be limited to times of crisis, such as now, and needs to be followed by a surplus to pay it back in preparation for the next crisis). If you are going to rely on borrowing like this (and most countries do - there are very few than run consistent surpluses) then you need to maintain a good credit rating, which means not defaulting on your debts (even though there is no law stopping you from doing so, since you make the laws!). As for the UK, our debt is actually still fairly low compared to other countries in Europe (although it's expected to increase dramatically over the next couple of years). The UK government has been sticking to a target of 40% of GDP (it's failed to stay within that recently, of course), the EU rules are 60% and some major EU countries haven't even managed to stick within that prior to the current crisis (and will presumably go well over now). The UK is going to take on massive debt and that's not good news, but I see no reason to believe it will be unable to service that debt (no-one is looking forward to the tax rises that are going to be required to pay it back in a few years time, but we'll manage). I haven't read the article yet (I will do now), but I don't see how joining the Euro would make any difference (I doubt it's even practical to have such a major upheaval during a recession). --Tango (talk) 15:38, 29 November 2008 (UTC)[reply]
The negatives of running a structural deficit (as opposed to only having a deficit when absolutely necessary, like during WW2 or as a result of bailouts for the current financial crisis) are many:
1) Not only must the money borrowed be paid off by future generations, eventually, but the interest must be paid, too. This means that future generations will need to pay more than the current generation gets. Only if the interest rate was less than inflation would this not be the case, and, if that was true, people would be foolish to loan the government money. There is a "taxation without representation" argument on saddling future generations, who have no vote, with our debts.
2) If we always borrow close to the limit of what will cause economic collapse, then we can't borrow much more when we really need it, such as during wars and the current financial crisis.
3) Being dependent on lenders such as China puts us at a political disadvantage, as they can threaten to withhold future loans if we complain, say, when they invade Taiwan or have another Tiananmen Square Massacre. We can, of course, threaten not to pay our current loans to them, but that would make others even less likely to lend us money. StuRat (talk) 17:34, 29 November 2008 (UTC)[reply]
I take it 'we' and 'us' refer to the United States here? Algebraist 17:38, 29 November 2008 (UTC)[reply]
No, it means any Western nation which borrows heavily from others. StuRat (talk) 17:42, 29 November 2008 (UTC)[reply]
1) The interest must be paid, certainly, but there is no need to ever pay back the principle. You just take out new debt to pay off the old. As long as the economy continues to grow fast enough that interest payments as a percentage of GDP do not become too high, there is no problem (if you hit a recession, you have a problem, but it's just a temporary one - once you come out the other side you have to pay back some of the debt to get back to affordable levels, but after that you can continue as you were). 2) Absolutely, I believe I said as much. 3) There is a problem when a foreign country owns a significant amount of your debt, it's true. Ideally most of your debt is held domestically and the rest is mostly spread between friendly nations. --Tango (talk) 18:54, 29 November 2008 (UTC)[reply]
Two comments on point #1: First, the interest (minus the inflation rate) on a perpetual loan will eventually total more than the principal. Second, I still say that the principal will either one day need to be paid or the country will be in default. A global depression, for example, will eventual occur so that the money is no longer available to borrow in order to "rollover" the debt, meaning the principal will need to be paid off under the worst possible circumstances. StuRat (talk) 19:15, 29 November 2008 (UTC)[reply]
I don't understand your first point, what is the significance of the total interest paid becoming greater than the principal? As for your second point, in a depression there is less money to go around, certainly, but people will generally want to lend what money they have to (reliable) governments since it is safer than investing in anything else (barring gold, perhaps) during a time when businesses are struggling to survive. Have governments had problems borrowing money in depressions in the past? (I haven't studied the subject in any depth.) --Tango (talk) 19:44, 29 November 2008 (UTC)[reply]
My point is that, while it's immoral to borrow $1 and make your grandchildren pay back $1 (adjusted for inflation), it's both immoral and stupid economics to borrow $1 and make your grandchildren pay back $30 (adjusted for inflation). I don't believe we had anywhere near the level of debt we have now, before the Great Depression, so borrowing money wasn't quite the issue it is now. For those people with money in a depression, there are many fantastic opportunities, like buying up failed companies and cheap prime real estate, so government bonds would be a rather poor choice. StuRat (talk) 04:17, 30 November 2008 (UTC)[reply]
In order to compare cash flows at different times, you don't want to adjust for inflation, you need to discount by the risk-free rate, which is generally taken to be the interest rate of government bonds. If you do that then I think you end up with the amount being paid back being exactly equal to the amount borrowed in the first place. (I'm assuming the risk-free rate is constant, which isn't true, but it will do for a rough impression.) Buying up failed companies is only good if you're an expert on rescuing failed companies, most people aren't. Buying up cheap real estate is only good if you are happy to wait a long time for the depression to end and property values to increase, people that want more short term investments will and do turn to treasury bonds. --Tango (talk) 13:44, 30 November 2008 (UTC)[reply]
I don't get your argument at all. If money is borrowed, and interest is paid on it continuously (over the rate of inflation), and the principal is never paid off, then the portion of those interest payments above inflation is money the government loses each year. The only way this could be good economics in the long run is if the government invests that principal in some way that grows faster than the interest rate minus inflation. And government rarely makes good investments. StuRat (talk) 16:57, 30 November 2008 (UTC)[reply]
That investment is the economy, if economic growth is greater than interest on government bonds minus inflation (or, not minus inflation, depending on how you measure growth) then even if you pay the interest by borrowing more, your debt as a percentage of GDP is still decreasing (allowing you to borrow more to cover a budget deficit). It's when economic growth drops (such as now) that you have a problem, but over a long enough time scale you can be pretty confident it will average out to a decent growth rate. --Tango (talk) 17:19, 30 November 2008 (UTC)[reply]
I challenge the assumption that borrowed money is wisely invested to cause the economy to grow. If the government is limited to only spending money it actually has, then it's apt to make wiser decisions about where to spend that money than if there is an unlimited supply of money available due to heavy borrowing. An "open treasure chest" leads to wasteful spending. StuRat (talk) 19:12, 30 November 2008 (UTC)[reply]

(Rm indent)

@Tango: I think another way to illustrate the theoretical problem of future generations funding current expenses is that the future generations bear the risk of rolling over the debt at higher interest rates. If the current administration stuffs up (gets downgraded for example) then future administrations have to bear the burden. Of course, the current generation can also improve the situation and leave the future generation better off but the fact is that there is always the risk that it might get worse.

@Sturat: I don't believe much of the spending is wasteful wrt to future generations. It can even be argued that the money spent on wars leave future generations better off. The problem is that the future generations aren't involved in the decision making but still bear the risk as explained above. Zain Ebrahim (talk) 06:28, 1 December 2008 (UTC)[reply]

Part of the debate turns on the definition of debt used. The figures quoted above exclude various liabilities which might reasonably be included, such as the Private Finance Initiative. The largest government liability excluded from official figures is the enormous bill for unfunded public sector pensions. The government supposes that this liability amounts to GBP 650 billion, broadly the same as the official figure for debt, but others suggest higher figures. The Institute of Economic Affairs are quoted as coming up with a figure of GBP 1200 billion or thereabouts. So, yes, the official figure for UK debt is relatively low, but there are good grounds for thinking that it is a nonsense. Angus McLellan (Talk) 16:00, 29 November 2008 (UTC)[reply]
Do other countries not fiddle the books in the same way, though? --Tango (talk) 16:03, 29 November 2008 (UTC)[reply]
Yes, the US certainly does. The party in power always says the debt is tiny and the opposing party says it's huge. StuRat (talk) 17:45, 29 November 2008 (UTC)[reply]
Putting aside the theoretical discussion of government debt, which I think Tango has covered quite well, and turning to the present case of the UK, as discussed in the Guardian article, I believe that there is indeed some risk that the United Kingdom (and ultimately the United States, for that matter) might eventually face a debt and currency crisis. The reason why this is a real possibility is that the debts of the United Kingdom (like those of the United States) have been expanding at a dramatic and accelerating rate as the government rushes to bail out an enormous and disastrously insolvent financial sector. Because the British financial sector is larger as a proportion of the total economy (GDP) than that of the United States, the government's debt-financed assumption of the financial sector's bad debts threatens investors' faith in the ability of the British government to repay its debts.
While the debt of the UK as a percentage of GDP is currently modest by comparison to other developed nations, the scale of the financial sector's insolvency is unknown and could amount to trillions of pounds. If the British government were to try to finance this, it would alarm investors. Apparently, investors are already alarmed, because they have been selling sterling assets at such a rate that sterling has lost a quarter of its value in just three months. If investors were to lose all confidence in the creditworthiness of the UK, its currency could plummet and/or become nonconvertible, as the currency of Iceland did for a few weeks recently. The only way for the government to fund its debts might be for the Bank of England to print pound notes, or their electronic equivalents, and use these to buy government bonds. This would further unnerve holders of sterling assets, who would see their value diluted by the expansion of the money supply without corresponding economic growth. This would add to the downward pressure on sterling and would lead to sharply higher prices for imported goods and a risk of destabilizing inflation. None of this is certain to happen, it must be emphasized, since no one can predict the future.
The author of the Guardian article argues that it would be prudent for the UK to adopt the euro now as a way to avoid a run on sterling. If the UK were to join the euro, the scale of the euro zone's financial insolvency would probably be much smaller relative to that of the total economy, since the financial sectors of the euro zone's other economic powerhouses, Germany and France, are much smaller in relative terms. Investors are less likely to be anxious about the solvency of the euro zone as a whole, especially when the euro's main competitor, the US dollar, somewhat like the pound sterling, faces a greater relative risk of a crisis of confidence.
What the article's author neglects to point out is that EU members seeking to adopt the euro are expected to participate in the European Exchange Rate Mechanism for two years before adopting the euro. That is, they are required to keep their currency closely pegged to the euro, generally by limiting their budget deficits and raising interest rates to limit current account deficits. These policies would make it impossible for the British government to continue its bailout of its financial sector, or to attempt economic stimulus on a larger scale than the less-stricken euro-zone countries. So the author's suggestion that adopting the euro would be an easy way out is misleading. Marco polo (talk) 23:06, 29 November 2008 (UTC)[reply]
(ec with your last paragraph) I think what you describe is unlikely, but it certainly isn't impossible. However, joining the Euro wouldn't help - if the UK government's creditworthiness drops to levels where it cannot reasonably borrow money, and it can't print money since it has surrendered monetary policy, what would it do? It could raise taxes up to a point, but it would quickly reach the peak of the Laffer curve, and after that it would be left with no choice but the dramatically cut public spending. Joining the Euro would increase confidence in the currency, but that doesn't mean there would be confidence in the UK government's ability to pay its debts. At the moment, the yield on UK 6-month government bonds is 1.14% [7],compared to Germany's 1.93% and the US's 0.42% (same site), which suggests to me that investors are pretty confident in the UK government's ability to pay its debt in the short term (the yields also factor in the risk of other investments in that currency, so it's not a perfect measure - I'll see if I can find some credit default swap numbers, which would be better). --Tango (talk) 23:28, 29 November 2008 (UTC)[reply]
Tango, I think that the low yields on short-term bonds in the US and UK do not so much reflect investors' confidence in governments was they do investors' lack of confidence in other asset classes and the rate at which the Federal Reserve and the Bank of England are flooding their systems with liquidity. All of that cash has to go somewhere. According to this article, the UK money supply is expanding at an annual rate of 15%. According to this source, US M3 (broad money supply) is expanding at a year-over-year rate of well over 10% (albeit decelerating). Meanwhile, according to this source, euro-zone M3 is growing at "only" 8.7% (admittedly still quite high relative to economic growth). Meanwhile, the discount rate is now higher in the euro zone than in either the US or the UK. Investors need some compensation for holding a bond rather than cash. They need more compensation for a German bond because of the higher interest available for a euro cash investment. The only way that I can explain bond interest rates so much lower than interest rates on bank accounts is the fear of people and institutions with high net worth of depositing funds in excess of government guarantees in possibly failing banks. Maybe that fear is greater in the United States and the United Kingdom than in Germany. Of course, this is speculation, but in the world of finance, especially today, there is much that is not transparent. Marco polo (talk) 02:15, 30 November 2008 (UTC)[reply]
Indeed. Credit default swaps are a far better way of comparing percieved risk of default, but I can't find prices for them (I found a price of 88 basis points for UK debt on the 24th Nov in a Bloomberg article, but couldn't find anything to compare it to). However, I wasn't meaning to suggest that them being low meant they were percieved as low risk, but rather that the UK yields weren't higher than the German yields which is what one would expect if the UK were a higher risk than the Euro zone (it could just mean that private UK investments are a higher risk relative to UK government debt than private German invests are relative to German government debt, though, which is why it's all difficult to compare). --Tango (talk) 13:44, 30 November 2008 (UTC)[reply]
Given the number of questions about national bankruptcy, should we start an article? Warofdreams talk 00:46, 2 December 2008 (UTC)[reply]

Pakistan

Is this true that all four major ethnic groups of Pakistan can have followers of Hanafi, Hanbali, Shafi'i, Maliki, Ithna Ash'ariyah, Ismaili Mustali Sulaimani Bohra, Ismaili Mustali Dawoodi Bohra, Barelvi, Deobandi? but for the Khoja Ismaili Nizari, all I know is that they are Sindhi. Which province have the most population of Christians? Which province has the most population of Hindus? —Preceding unsigned comment added by 76.64.53.184 (talk) 15:29, 29 November 2008 (UTC)[reply]

Sounds like something which would demand sustained library research (possibly necessitating a knowledge of Urdu) to answer very well. By the way, I suspect that some Deobandis and Barelvis might possibly take issue if you insinuated that they were outside the four madhahib.... AnonMoos (talk) 17:21, 29 November 2008 (UTC)[reply]

That doesn't answer my question about the article.

Bad Company song

I was making out with a girl last night in her bedroom, and in the background she put on some music so other people wouldn't hear; there was a song by the band Bad Company, and the song mentioned the band name Bad Company in the actual lyrics. Which song is this?-Norman the Doorman (talk) 16:59, 29 November 2008 (UTC)[reply]

This is really a question for the entertainment desk, but could it be that the lyrics "Bad Company" were from the song Bad Company from the album Bad Company by the band Bad Company? Talk about self references! Fribbler (talk) 17:06, 29 November 2008 (UTC)[reply]
The girl sounds enthralling.NByz (talk) 23:24, 29 November 2008 (UTC)[reply]
Holy crap. Are you 45? Do people still make out to "Bad Company"? --Jayron32.talk.contribs 02:05, 30 November 2008 (UTC)[reply]
Why not? Would you rather make out to Fall Out Boy? Adam Bishop (talk) 06:12, 30 November 2008 (UTC)[reply]
Gawd no, I found that Closer is the best makeout song ever. The chorus generally gets you where you want to go... --Jayron32.talk.contribs 20:13, 30 November 2008 (UTC)[reply]
No, I'm 22. The girl was about the same age as me, actually slightly younger. And plus she is from Eastern Europe so maybe her country is kinda behind with music a little, but I like Bad Co. too, or at least what I've heard. Hey, I told her I liked My Chemical Romance as well and that nearly puts her off me completely. And I already made out to that particular Nine Inch Nails song with another girl like 3 years ago but this girl is a lot better. Not fat like that other chick back then. And Bad Company is cooler than NIN, even though I like the latter as well.-Norman the Doorman (talk) 21:36, 30 November 2008 (UTC)[reply]
Do you know that I cannot recall a single time in mumbledy-mumble decades of active sex life where the music made a difference, before or during. Is this just me personally, or is it a male-female thing? I think all of the preceding remarks have been made by males. ៛ Bielle (talk) 05:20, 1 December 2008 (UTC)[reply]
Well, as much as I appreciate the music of John Philip Sousa, it would make a rather odd choice in my sensitivity, so I guess I do think the music can make a difference (but then I'm male ;) ---Sluzzelin talk 09:55, 1 December 2008 (UTC)[reply]
Heh, I can just about imagine John Cleese or Michael Palin bonking to Liberty Bell. But then, I can hardly imagine them doing anything at all except in a bizarre way. -- JackofOz (talk) 20:31, 1 December 2008 (UTC)[reply]
As anyone who has watched Fast Times At Ridgemont High knows, "when it comes down to making out, whenever possible, put on side 1 of Led Zeppelin IV". Hammer Raccoon (talk) 15:15, 2 December 2008 (UTC)[reply]
Bielle, I'm with you (though I only dimly remember my active sex life). —Tamfang (talk) 21:19, 4 December 2008 (UTC)[reply]

Where does stereotypical 'gay' behavior originate?

I was curious where stereotypical gay mannerisms and speech originate from? Is the speech and mannerisms a learned behavior? If so, what are the sources? It seems to be completely unique and easily identifiable across ethnic, linguistic and cultural borders. This isn't questioning of homosexuality. But there are friends I know who are referred to as "screaming queens" and others who aren't. They are usually referred to as "effeminate", but I don't see that type of speech and mannerisms as trying to replicate a woman. --70.130.54.91 (talk) 18:26, 29 November 2008 (UTC)[reply]

One clue is that it was once fashionable for upper-class men to behave like that, wearing high-heeled shoes, wigs, and perfume, and talking/walking in an effeminate manner, even when they were straight. See fop and dandy. Then there's the modern equivalent, the metrosexual. StuRat (talk) 18:33, 29 November 2008 (UTC)[reply]
Just because women don't actually act effeminate in the same way doesn't mean that it's not meant to be in reference to some projected idea of that. I find it very interesting how gay males act in a way about what it means to be "feminine" which is more about an inversion of "masculine" than it is anything that women actually do. Similarly I find the "butch" variety of lesbians do very similar things—taking on various "masculine" characteristics that are really extreme projections of the idea of "masculinity" (in particular the uneducated, brutish form). (On the latter point I've always wondered why one would want to do that, as a non-particularly "macho" man I have found the hyper-masculine, thuggish sort of attitude always reprehensible and find it so very odd that lesbians would embrace it for any reason, since it seems entirely opposite to the values they embrace, though I understand these things about identity are complicated.) --98.217.8.46 (talk) 19:03, 29 November 2008 (UTC)[reply]
I read your question as whether stereotypical gay or lesbian behaviour is genetically or culturally transmitted. I can't answer that question, but acknowledge the fact that that an autosomal gene that predisposes to behaviour that attracts male sexual partners, might have an evolutionary advantage, i.e. that the advantage in female carriers might outweigh the disadvantage in male carriers. If anyone has references to studies that address this question, it would be most welcome. --NorwegianBlue talk 22:50, 29 November 2008 (UTC)[reply]
(after ec) Pick a movie from the 1940s, anything featuring, for example, Joan Crawford, Veronica Lake,Myrna Loy, Mae West or Bette Davis. Watch them vamp their way throughout the movie. (See even the photo on the Crawford article.) These are the models, I believe, for the excruciatingly exaggerated mannerisms for the sexy female. (This is WP:OR through and through, by the way.) I think that when stage actors first came to the movies, they brought with them the same overdone gestures that were useful on stage when the audience might be too far away to see facial expressions or small body movements. Everything from smiles to ambulation had to be "visible" from 15 metres. It took quite some time for the nuances of the big screen to be exploited. And thus, at the time when movies had the most inflence on popular culture, they were displaying these grossly overdone caricatures of female presentation as indicative of star quality. I think that is where the stereotype came from, but as to why it is supposedly worth being copied, I can only speculate that it is its very exaggeration that is its attraction: more female than a female, as it were. ៛ Bielle (talk) 23:13, 29 November 2008 (UTC)[reply]

i'm sure its about emulating women actually.. i'm gay and on the camp scale, whilst not a 'screaming queen', i am what you would call 'obvioulsy gay', like I imagine graham norton is off-camera. anyway, i think i started adopting gay mannerisms after meeting my first 'real-life' gay and, whilst basically falling in love with him, thought he'd love me back if i was like him..(he didn't). at that time i regarded this type of self-expression as reaction (in a positive sense) against a conservaive upbringing and as such saw it as an integral part of me (and, to a lesser extent, i still do).


Thats my point of view anyway. If you were to ask, say, my (elder) sisters or some of their friends, they would tell you instantly that i was clearly gay from an early age (someone claims to have outed me when i was 12, whilst i outed myself at age 17/18). If true, this makes me think that perhaps i was only exaggerating what was already there. (Which ties in with the earlier evolutionairy point made earlier). Certainly i was the stereotypical gay child (hopeless at sport etc) but there were plently of others like that at my school who are now resoundly heterosexual, so lets keep this scientific....


I guess my main concern about the 'emulating women' argument is that its quite hetero-centric (the corrollary being that there are woman-gays and man-gays). Lets not assume heterosexual norms are still universally valid for homosexual realtionships!82.22.4.63 (talk) 11:01, 1 December 2008 (UTC)[reply]

OK, you seem to be the resident expert, so here's some follow-up Q's for you:
1) Does it attract you if another man acts "stereotypically gay", with the lisping voice and such ?
2) Does it make it easier for you to identify them as homosexual ?
3) Do you believe you attract men by acting that way yourself (men who would not be attracted to you otherwise) ?
4) Is it all "an act" you put on for others, or would you talk like that when conversing with, say, a dog, with no witnesses ?
5) Could you act convincingly straight if you wanted to ? StuRat (talk) 13:42, 1 December 2008 (UTC)[reply]


Oooh the 'resident expert' - i like it! Here goes with some answers... 1) actually i don't think it affects me. There is certainly some considerable anti-camp predjudice in the gay community but i don't seem to roll that way.. (i've been attracted to some camp guys and some non-camp guys)

2)yes, it makes it 'easier', in the same way that crossing the road is 'easier' if there is less traffic, but usually its not difficult to tell if someone is on 'your team' or not. (intra-gay 'gaydar' seems to be quite accurate -there are doubtless more factors at play than just campness).

3)i repulse men so either way i couldn't comment

4) i would say on balance, no. like most people i do nuance my behaviour according to my surroundings. i do not, for example, act the same in a job interview as i do at a party. Having said that, i would hazard that most people who interview me would have an idea that i am gay... so yes -i would talk that way to a dog!

5)i take issue with the term 'act straight' (becuase it implies that gay and camp are synonymous - which they're not (see Brian Paddick/David Beckham)). anyway, to answer the question, no. i've tried both consciously (as a bet at a party) and sub-consciously (whilst growing up) and alas it can't be done (at least by me!).

That was fun but where does it leave us? 82.22.4.63 (talk) 16:18, 1 December 2008 (UTC)[reply]

I've heard of men saying they are a "straight-acting gay", so my question number 5 seems reasonable, to me. It seems from your responses that you don't really benefit from "camp" style, and can't convincingly change, so that implies that it's not so much a choice to act that way as it is "just part of who you are". Perhaps there's a genetic component to "acting camp" which is linked with male homosexuality, but not exclusively so. For comparison, many blonds have blue eyes, but there are also those with only blue eyes (like me) or only blond hair, so this is the same type of weak genetic correlation. StuRat (talk) 00:11, 2 December 2008 (UTC)[reply]
I'm not sure stereotypical gay behavior is identifiable across cultures because gender norms are different from one culture to the next. What stereotypical gay behavior is, at least in this discussion, is the refusal to adhere to what is masculine for men and feminine for women. Not all gay people behave this way. There are gay men who are hypermasculine and profess to dislike twinks and effeminate men. Similarly, there are very feminine lesbians who refuse to have anything to do with butch women. For some gay people their behavior is noticeably more gay when they come out, are younger, and are near other gay people. This may wane as one's identity expands beyond sexuality. I think that is essential to the original question: behavior is an expression of identity, or what one finds admirable. For a man to admire femininity and emulate it instead of desire it is not widely seen or accepted, therefore a subset of behavior that sets some gay men apart. Women who express strength in masculine ways are similarly unique. --Moni3 (talk) 00:57, 2 December 2008 (UTC)[reply]
I don't know -- I think it's pretty easy to spot "a gay" if you travel overseas. Even if I don't know the language or the culture, identifying an openly gay man isn't that hard. "They" all tend to exhibit similar mannerisms. (I don't mean 'they' in a dorogatory meaning, mind you.) So if it is learned behavior, where are they all learning it from? Or are social concepts of masculinity (and therefore anti-masculinity) universal across societies? --70.167.58.6 (talk) 06:22, 5 December 2008 (UTC)[reply]

Womens brains have higher verbal skills encoded in them , as opposed to more visual-spacial skills in the male. These verbal skills give them an almost musical voice, and indeed some studies show that the male responds in a similar way to hearing music. This is to be contrasted with the monotonous speech of the male, particularly in the case of "nerds" (such as myself) or extreme male brain types. Most gay men have a more "female" brain (due to hormonal effects during conception) and this musical speech pattern charcteristic of the female can be detected in them, probably one of the more noticeable characteristics of some gay men, even non-effeminate looking ones.Trevor Loughlin (talk) 04:38, 2 December 2008 (UTC)[reply]


Thats really interesting (about musical voices) and certainly seems to apply to a number (but by no means all) of my gay friends. StuRat, i think your analogy to blond hair and blue eyes is a valid one. You are also right in that straight acting is used by the gay community but i still find it hideous and, when used by other gay men, to be indicative of the self-hate that sadly affects our community. Its a personal thing of course but these things always are..82.22.4.63 (talk) 16:18, 2 December 2008 (UTC)[reply]

European citizen outside the EU

Can European citizen use the diplomatic representation of any European country abroad?--Mr.K. (talk) 21:06, 29 November 2008 (UTC)[reply]

Yes, if by a "European country" you mean a European Union member state and if the EU citizen's home country does not have its own diplomatic or consular representation in the country where the person is travelling. According to Article 8c of the Treaty Establishing the European Community:
Kpalion(talk) 22:22, 29 November 2008 (UTC)[reply]
The answer is no you can't. Only if you are a European citizen and there is no diplomatic mission of your country in that country's territory you may use the diplomatic mission of another European Union Member State. My parents were on holidays in Sri Lanka when the tsunami caused by the 2004 Indian Ocean earthquake hit, they got away with their lives but lost all of their money, documents and most of their belongings. The nearest Polish diplomatic mission was in Mumbai, India, but my parents they got the same help as the French citizens in the French Embassy in Colombo. Mieciu K (talk) 22:34, 30 November 2008 (UTC)[reply]
They are only obliged to help you if there isn't a consulate of your country in the country, but do you know if they would help you even if there was a consulate in the country, just not one nearby or accessible? For example, a war breaks out and you can't get from one side of the city to another, could you just turn up on the doorstep of an EU embassy that's nearby? I would expect so, simply out of humanitarian concerns. --Tango (talk) 00:03, 1 December 2008 (UTC)[reply]
"in which the Member State of which he is a national is not represented" the literal interpretation of this text implies (to express indirectly) that under this specific law they do not have to help if there is "his" diplomatic mission in the same country. You do not have to go to "your" embassy in person usually a phone call is enough for them to organize some sort of help for you. Another story is when "your" embassy refuses to help you. I have recently read a story about a Czech citizen getting scammed out of his money and wanting to return to the Czech republic. The Czech consulate refused to help (citing that they don't help irresponsible people) so the Polish Police officers started a private fund drive and bought this guy a ticket back home. Mieciu K (talk) 00:35, 1 December 2008 (UTC)[reply]
Maybe the Czech consular service aren't as nice as the UK one, but my understanding was that your consulate would normally put you on a flight home in that situation and then bill you when you got back home. --Tango (talk) 13:55, 1 December 2008 (UTC)[reply]
Of course if you aren't in an EU country you'll be sent back by the local authorities anyway once you no longer have the legal right to stay in that country (e.g. once your visa runs out). You may also be detained, fined or even jailed so it's not necessarily a great idea. And yes, you will be billed once you get home Nil Einne (talk) 14:11, 1 December 2008 (UTC)[reply]


November 30

Barcode Tattoo Scenario

What are the chances of the Barcode Tattoo scenario coming true. I know I should be concerned about being implanted with a micorchip but do I have to worry about it happening? Should I be a strong activist against it or be comfortable knowing it's very unlikely it will happen? --Melab±1[[User_talk:Melab-1|☎ 03:08, 30 November 2008 (UTC)[reply]

Your best defense against this occuring is to wear a tinfoil hat... --Jayron32.talk.contribs 03:13, 30 November 2008 (UTC)[reply]
The government just loves to spy on us, but one of the prerequisites for spying is that the subjects be unaware of the surveillance. Since it would be literally painfully obvious when they tattoed you with a bar code, this would cause people to vote out any politician which voted for such a thing. We would need to have democracy abolished before such a program could be forced upon us. And, since Dick Cheney didn't run for President, there's no chance of that happening any time soon. StuRat (talk) 04:08, 30 November 2008 (UTC)[reply]
See Mark of the Beast. Little Red Riding Hoodtalk 05:45, 30 November 2008 (UTC)[reply]
You could look at the book Database Nation written in the late 1990's. No tattoos needed, all that info can instead be stored on remote computers. Consider a V for Vendetta Guy Fawkes mask for when you go out in public, to defeat being recognized by the surveillance cameras that are already everywhere. Who needs RFID? 67.122.210.149 (talk) 05:50, 30 November 2008 (UTC)[reply]
Do you carry a cell phone? If so, anybody who really wants to can know where you are. -Arch dude (talk) 02:28, 1 December 2008 (UTC)[reply]

The respondents so far seem to be treating this in a fairly light-hearted manner and I'm not sure that's helpful. Any politician that took the platform of "Hey guys, we'd like to microchip you and maybe slap a UPC on your neck just for show!" would obviously get drummed out of office pretty quickly, but I don't think that's the route an attempt at tracking would take. As StuRat says, it's not really spying if everyone knows about it beforehand... or is it? Our cellphones are equipped with GPS technology so our positions can be targeted at all times. You're under video surveillance every time you enter a retail store or buy gas and sometimes even when you go out to dine (especially fast food). If you drive on toll roads, your license plate is captured when you get on and when you get off. Unless you're using that old "cash" thing, most financial transactions you make are recorded down to the minute of purchase. If you buy goods at a "club" store (i.e. where you need to be a member), you'll get a call if one of the items you bought was involved with a food safety recall. We appreciate the safety and convenience, but seldom consider the downside. Will we be forced to get micro-chipped? Nah, not in the very near future. On the other hand, many of the children in my kid's school have already been finger-printed (to "help track them down in case of them getting lost or abducted"), which is basically only one step away. Many people already chip their pets (with short distance RFIDs). I would not be at all surprised if the next generation chips their children in the name of "safety". In other words, "Yes, you should be worried about it happening, but beware that we're already partway there - of our own choice." Matt Deres (talk) 15:45, 1 December 2008 (UTC)[reply]

Yes but I mean like actually being chipped. It's one thing to be followed by satellite but it's another to have someone (gov't) force you to chipped. And I'm asking is our gov't and others like UK, Germany, France, Canada, and the EU level headed enough not to do so. --Melab±1 23:50, 1 December 2008 (UTC)[reply]
Who keeps copies of the fingerprints? The schools or the parents? If only the parents have copies, I don't think that's too big of a problem. 216.239.234.196 (talk) 18:22, 1 December 2008 (UTC)[reply]
I should point out a number of countries already have, and have had for a while, finger prints as part of a national identity document. Also many mobiles still don't have GPS although this is likely to change over time and you can track phones resonably well without GPS. And BTW, people are already RFID their kids [8] (whether any of these are implants or not, I haven't found out, I suspect not though since most parents interested quickly realise it's somewhat useless. They'd much rather have a trackable chip and we can't achieve that in implants yet) see also Microchip implant (human) Nil Einne (talk) 21:12, 1 December 2008 (UTC)[reply]

Person mentioned in Letters to Olga

Who is/was Andulka? Havel mentions her several times, but there's no explanation of who she is in the notes about the names mentioned in the back. Vltava 68 (contribs) 12:14, 30 November 2008 (UTC)[reply]

There is a passage where Havel asks Olga to congratulate and greet "Andrej and Andulka" and asks what they will call their daughter (Letter to Olga dated August 13, 1979). One of the Havels' good friends was the theater director Andrej Krob. Maybe Andulka was his wife, but I found no information confirming this. Just a guess. ---Sluzzelin talk 16:12, 30 November 2008 (UTC)[reply]
Which copy is this? The letter for that date in the copy I'm reading doesn't mention anything like that. --Vltava 68 (contribs) 09:08, 1 December 2008 (UTC)[reply]
Well, perhaps it's dubious, Vitava, sorry. I saw it in a French lesson on epistolary literature on "Académie de Versaille"'s website. here, search for "Andulka". On that site, it is dated "Lundi 13.8.79" I assumed the date and content were correct, but of course this isn't what is considered a reliable source at all. ---Sluzzelin talk 10:11, 1 December 2008 (UTC)[reply]

Hungarian language query: Dolgozók vs Munkás

What would be the difference in generic meaning between Magyar Dolgozók Pártja and Magyarországi Munkáspárt? At present one is at Hungarian Workers Party and one at Hungarian Labour Party, but I think that difference might be rather arbitrary. Perhaps one should be 'Toilers' or 'of Labour'. Also, whats the difference between Magyar and Magyarországi? --Soman (talk) 15:08, 30 November 2008 (UTC)[reply]

This should be on the Language Ref Desk. StuRat (talk) 16:40, 30 November 2008 (UTC)[reply]
Shifted there now. --Soman (talk) 19:15, 30 November 2008 (UTC)[reply]

Authenticity of a history doctrate

I'm aware of my ex brother in law referring to himself as a 'doctor' of history.Is it possible to find out whether this is a genuine qualification?

Probably. Whether university documents would be public access info depends from country you live in. The easiest is probably to call up the historical faculty of the university in question. --Soman (talk) 15:35, 30 November 2008 (UTC)[reply]
A doctorate in history is a Ph.D. in history. Call up the department or university and they'll tell you, it's all on records. Depending on the university it might eve be online, as dissertations are usually listed in their library catalogues. --98.217.8.46 (talk) 16:25, 30 November 2008 (UTC)[reply]
I always thought that you have to publish your doctorate dissertation. If this holds true, you could just search for this publication.--Mr.K. (talk) 18:11, 30 November 2008 (UTC)[reply]
UMI indexes and sells copies of almost all doctoral dissertations submitted to U.S. institutions... AnonMoos (talk) 18:34, 30 November 2008 (UTC)[reply]
Or anyone with access to ProQuest could look it up. But again, you almost always have one in the home institution library system, which is an easier way than trying to look up the dissertation itself if you don't have easy access to tools that do that. (Academics do, but most other people don't.) --98.217.8.46 (talk) 20:03, 30 November 2008 (UTC)[reply]
You don't have to publish your dissertation, but most people re-work it into a book or break it up into a bunch of articles. Otherwise you just waste a lot of time and money! (People sometimes disown their embarrassing BA or MA dissertations but for a PhD it's harder to get away with writing a bunch of crap.) Adam Bishop (talk) 02:07, 1 December 2008 (UTC)[reply]
I've never heard a person with a Ph.D. in X say, "I'm a doctor of X". Are you sure he didn't say he was "a student of history"? --Sean 20:38, 1 December 2008 (UTC)[reply]
Being a "doctor of X" suggests a higher doctorate to me - but higher doctorates aren't usually given in history (except, it seems, in the Soviet Union), and if your brother-in-law had a higher doctorate, you'd probably know about his distinguished career. Warofdreams talk 20:57, 4 December 2008 (UTC)[reply]

Having a Ph.D. means that you are a "doctor" of the philosophy involved in the study of "X," to begin with. Also, no, receiving a Ph.D. is not dependent upon publication of your dissertation, thank god.

How to deal with weasels?

And please don´t point me to How to Win Friends and Influence People, since I don´t want to make friends.--Mr.K. (talk) 18:05, 30 November 2008 (UTC)[reply]

Stock up on horned owls, martens and wolves. ៛ Bielle (talk) 18:38, 30 November 2008 (UTC)[reply]
Or you can badger them or rat them out. :-) StuRat (talk) 19:05, 30 November 2008 (UTC)[reply]
Don't forget about the "influence people" part. What precisely is the issue and what exactly do you want to accomplish ? StuRat (talk) 19:05, 30 November 2008 (UTC)[reply]
For anyone who doesn't want to win friends, but give their spin doctor the sack, there's the spin-spotting tool, weasel words. Julia Rossi (talk) 10:47, 1 December 2008 (UTC)[reply]
First make sure it's a weasel you're dealing with, not a stoat. It's easy to tell them apart. A weasel is weasily wecognised, while a stoat is stoatally different. --Richardrj talk email 10:56, 1 December 2008 (UTC)[reply]
Yes, they are weasels. Making "innocent" comments that serve no other purpose than to cause discomfort to someone, believing that they are much more competent than others, watching for mistakes of others, remembering everybody of said mistakes, selectively perceiving and questioning, and more, much more... The only useful advice -when the relationship is between co-workers- that I received until now is making their strategy the topic of conversation in front of them, since they are acting as if everything is business-as-usual and they have normally a long list of enemies. As employer is probably better just to fire these people. Mr.K. (talk) 16:55, 1 December 2008 (UTC)[reply]
Sounds like a stoat to me Nil Einne (talk) 21:05, 1 December 2008 (UTC)[reply]
I had a boss like that. Pretty much everyone who worked for him decided that avoidance was the best strategy. When we saw him coming we would all manage to be elsewhere. We all found that the less contact we had with him on a given day, the more we got done, and the better we felt. StuRat (talk) 23:57, 1 December 2008 (UTC)[reply]
Was he a weasel, a stoat, or a ferrit? Nil Einne (talk) 14:58, 6 December 2008 (UTC)[reply]
He liked to badger people, by constantly annoying all of us. StuRat (talk) 05:16, 7 December 2008 (UTC)[reply]

Marriage in the New Testament Bible

Where does it say that marriage must be sanctified in the New Testament? If the Bible say marriage must be sanctified, does it mean people must go to CHURCH to get marry. Sonic99 (talk) 20:23, 30 November 2008 (UTC)[reply]

It doesn't. The NT mentions the institution of marriage (see Christian views of marriage) but never specifies anything regarding the act of marriage. Our current concept of church marriages with liturgy was not developed in Europe until the 11th/12th centuries, but it was a while before they were universally adopted. Common law marriage was legally and religiously acceptable for quite some time (its legal status in England was abolished in the mid 18th Century, and lasted longer in Scotland, USA, and other countries). Gwinva (talk) 23:18, 30 November 2008 (UTC)[reply]
In early Christian times, if weddings were connected with a church at all, they would be celeebrated at the church door (not inside the church), and marriage was not usually considered a sacrament... AnonMoos (talk) 23:24, 30 November 2008 (UTC)[reply]
In our era, marriage, in almost all cultures, is both a religious and a civil matter. In many of the so-called western cultures, the religious aspect is fading, except by way of the rituals involved, and it is only the legal aspect that matters. If the civil authority says you are married, then, for the purpose of all civil matters (laws on child support, taxation and the like), you are. Some religious authorities may not accept this as a marriage under their doctrine, however. If that matters to you, then you will likely have both civil and religious involvement. Marriages in the province of Ontario, for example, no matter how profoundly religious the celebrant, the place, the ritual and the vows, still require a marriage licence from the civil authority and take from the ceremony, as proof of the civil contract, a properly witnessed Certificate of Marriage. Sometimes the solely civil marrriage also has a ceremony with a non-religious celebrant, licensed by the civil authorities, officiating. In Ontario, even this much is not required as the concept of common-law marriage is alive and well, and all the laws that apply to those married by ceremony equally apply to those who have lived together for a minimum period of time and so declare themselves to be married. As far as I can tell, the only need for a marriage licence and subsequent marriage certificate would be for jurisdictions in which one might find oneself where common-law unions are not recognized, or for evidence in some estate or other legal matter should one partner no longer be living, (or to please your mother.) :-) ៛ Bielle (talk) 23:41, 30 November 2008 (UTC)[reply]
People didn't go to church in Old Testament times (to get married, or to do anything else), though some close enough to Jerusalem might have gone to the temple. Genesis 2:24 does say that a man leaves his father and mother and clings to his wife -- an odd statement, since Adam had no father or mother. Matthew 19:6 has Jesus deprecating divorce (permitted under Mosaic law), though five verses later he concedes that not everyone can accept this.
In Roman Catholicism, there's no requirement that a marriage occur in a particular place; the core of the sacrament is the mutual consent of the two parties -- the officiating priest is present as a witness for the church, but he doesn't marry the couple; they marry each other.
Many people conflate religious marriage with civil marriage, especially here in the States. In fact the state of Maryland couldn't care less what religious denomination you adhere to, so long as there's a qualified person who will attest that you and your partner met the requirements of the state. This person might a judge, a minister, a priest, a rabbi, and so on. Similarly, the archdiocese of Baltimore couldn't care less what's on the records of the state of Maryland; the archdiocese concerns itself with whether the two parties met the requirements of the church. --- OtherDave (talk) 01:23, 1 December 2008 (UTC)[reply]
I'm not so sure about that last sentence: most dioceses won't marry you without a (civil) license, or begin proceedings for an annulment before there's a (civil) divorce. - Nunh-huh 02:14, 1 December 2008 (UTC)[reply]
My point was only that the diocese isn't interested in the state's definition of marriage; you're right that the diocese follows procedural instructions for the state (e.g., signing the certificate and sending it in). If you get divorced in Maryland, you're no longer married in the state's eyes. This has no effect on the archdiocese and its marriage tribunal, who consider themselves experts on whether you're still married from the church's point of view (you are). A civil divorce is in no way a requirement for a Catholic annulment, particularly since the church doesn't recognize divorce; there are thousands if not tens of thousands of Catholics with civil divorces who cannot have their religious marriages annulled. --- OtherDave (talk) 14:58, 1 December 2008 (UTC)[reply]
According to Annulment (Catholic Church) "An annulment from the Catholic Church is independent from obtaining a civil divorce, although before beginning a process before an Ecclesiastical Tribunal, it has to be clear that the marriage cannot be rebuilt." To me this sounds like commonly you'd need a civil divorce first otherwise the tribunal would not be convinced you aren't able to rebuild your marriage or may be concerned you aren't serious about the annulment if you haven't at least initiated the process for a civil divorce. This is supported by [9] in the US (well there is uncertainty about the reasons). Nil Einne (talk) 18:29, 1 December 2008 (UTC)[reply]
The "cannot be rebuilt" evidence could come in many forms. It's amusing to think of tribunals, which technically don't believe in divorce, using them as evidence in support of annulment. A civil divorce remains neither necessary (though possibly useful) nor sufficient. Every day of the week, marriage tribunals deny annulments to someone with a civil divorce and a partner who's remarried. The key question is the state of the parties on the day of the (religious) marriage. Bending somewhat to reality, tribunals have come up with "defect of discretion" -- legalese for "you didn't know what you were getting into that day." This allows the tribunal to find the original marriage was invalid (e.g., because you did not intend to remain married for life, or didn't really know that's what the church expects) -- the route used for Joseph Kennedy, Jr. Ironically, a decision by the Roman Rota (14 years after the civil divorce) held that Kennedy and his ex, Sheila Rausch, are still married in the eyes of the church. The reversal poses an obstacle to having his subsequent marriage in 1993 recognized by the church. --- OtherDave (talk) 14:36, 2 December 2008 (UTC)[reply]
I'm not denying the tribunals/Catholic church isn't stricter on annulments then divorves in most countries or that they don't recognise divorces, I would just expect that in many countries they would want you to be civilly divorced unless perhaps you have a very good reason for why you haven't. Other then the fact it may mean you marriage could be rebuilt (since if it can't why haven't you divorced?), it may create a whole bunch of legal problems for you and for them they likely don't want, and don't want you to have to deal with. The situation is the same with marriages I believe, in many countries they will require you to take care of the civil aspect or they won't marry you even though the civil aspect is irrelevant in religious terms. Obviously if there is an exceptional reason then they may ignore the requirement but I'm pretty sure it is a common requirement. While the Catholic church may have many odd aspects, they don't tend to completely ignore practical realities without (in their opinion) good reason. In this case, there's usually no harm to them to if they accept the practical/legal realities even if they don't agree with them, so they usually do. In other words, while obviously being divorced is never going to be sufficient and there will be exceptions, it seems likely to me that in the US, and I expect often in the rest of the world they will expect you to be civilly divorced. Nil Einne (talk) 14:58, 6 December 2008 (UTC)[reply]


December 1

More Fops Please

After reading the above question regarding 'gay' stereotypes--and subsequently reading both Fop and Dandy

I'm thoroughly confuzzled! Both pages use Beau Brummell and the Scarlet Pimpernel as prime examples of their referent.

What, if anything, is the difference?192.136.22.4 (talk) 01:58, 1 December 2008 (UTC)[reply]

There is also Macaroni (fashion). As far as I remember (from reading a book about the Regency period, the title and author of which I forget...) there isn't really a difference except time period. Fop and dandy (and macaroni) are names for the same time of thing in different generations. Adam Bishop (talk) 02:03, 1 December 2008 (UTC)[reply]
Not gay, but extravagant-on-purpose were the Zoot suits and their wearers – see also Metrosexual, among others and Bling. What they seem to have in common is vanity, disposable income and an affinity with the city. Julia Rossi (talk) 10:24, 1 December 2008 (UTC)[reply]
And believe it or not, we don't seem to have an article on the dark side of vanity, Pimp style, but mostly there's a touch of this[10] or this[11]. Julia Rossi (talk) 10:41, 1 December 2008 (UTC)[reply]
A recent BBC drama on the life of Beau Brummell [12] contrasted the bewigged, powdered, and painted fops who wore brightly coloured clothing and knee-britches in an 18th C style with the dandy style promoted by Brummell - no wig, no powder, trousers, and dark clothing. The two groups were depicted almost like the Mods and Rockers of the day - even down to brawling with each other in the street. Whilst I can't vouch for the authenticity of the street-fighting, the programme made clear the distinction between fop and dandy which I had previously thought were largely interchangeable terms as far as the Regency period was concerned. As opinion rather than verifiable fact, I would suggest that in contemporary usage describing a man of today as a fop would generally be perceived as pejorative - there is a suggestion of decadence, foolishness and effeminacy - whereas to call a man a dandy would be a more neutral term). Valiantis (talk) 02:40, 6 December 2008 (UTC)[reply]

Barbara Ehrenreich

Is that Barbara Ehrenreich on the cover of her book? Gridge (talk) 02:32, 1 December 2008 (UTC).[reply]

I don't think so. Her picture is on the back cover of that same book, in the lower left hand corner; long hair, different nose, slightly different facial proportions. The cover credit on my edition is "Gilles Peress/Magnum Photos" but does not identify the subject. Harrowing read, by the way. Antandrus (talk) 03:08, 1 December 2008 (UTC)[reply]
Thanks. Gridge (talk) 16:17, 1 December 2008 (UTC).[reply]

Senator's terms

On what day in January 2009 are the U.S. Senator's sworn in for their new terms of office? 24.29.246.224 (talk) 02:56, 1 December 2008 (UTC)[reply]

January 3 - which in 2009 will be a Saturday, I think; I believe unless it's on a Sunday, they are sworn in on that date.Somebody or his brother (talk) 03:00, 1 December 2008 (UTC)[reply]

Tousch

I've been doing some research into Alma Moodie for her article, and I came across the word "tousch", apparently meaning a kind of special acclamation at a concert, reserved only for the most worthy of performers. I'd never heard of "tousch", and all my searches have failed to come up with references to it. I've tried various spellings, but no luck. The exact context goes:

According to Reger’s wife Elsa, Reger sent Alma to Meiningen where he had indeed engaged her to play the Brahms concerto with the Meininger Hofkapelle on 9 December 1913. ... The same program was given at Eisenach on 6 December and Hildburgshausen [sic; it's spelled Hildburghausen] on 7 December. Now, her reception as a young prodigy virtuoso was unambiguously triumphant. At Meiningen, she wrote back to Louis D’Hage in Rockhampton, the members of the audience stood on their chairs and shouted ‘bravo’—this despite Reger’s warning that the audience was too aristocratic and the ladies too frightened of cracking their gloves to clap. Finally, the ‘tousch’ was given, a rare acclamation which, Moodie told D’Hage, had not been given since von Bülow’s time (thirty years earlier).

Any help anyone can give me with this would be much appreciated. Over to you. -- JackofOz (talk) 03:49, 1 December 2008 (UTC)[reply]

I swear I've heard the word being used as an acclamation, but I can't remember where that was. bibliomaniac15 03:51, 1 December 2008 (UTC)[reply]
There is the German term der [13] Tusch (I thought it to be onomatopoeic, but it stems from the French touche). This is basically a drum roll, but other instruments can be involved and is a marker for the audience that some highlight is about to happen / has just happened. A sort of orchestral exclamation mark. You may typically associate those with a circus performance when the band plays a Tusch at moments of some particularly tricky and spectacular points in an artistes performance. I have never heard it used in English, but this is a reference to a historic event in Germany, so it may have been used in the context. --Cookatoo.ergo.ZooM (talk) 07:45, 1 December 2008 (UTC)[reply]
The dictionaries I checked translate "Tusch" with fanfare or flourish. The German Wikipedia article on "Tusch" has interwikilinks to the English article on Sting (musical phrase) (but see also Sting (percussion)). I agree with Cookatoo, that a "Tusch" (in German) is usually one of two things:
---Sluzzelin talk 15:21, 1 December 2008 (UTC)[reply]
I see. Thanks for those bits of information, folks. So, what could this mean in the context? She played her concerto, then they started clapping, then they stood on their seats and shouted 'bravo', then came the Tusch. She'd have been returning to the stage to acknowledge the applause, leaving, returning, leaving ... until she left and didn't seem to be coming back. This might have been the audience's cue to let her know that they weren't finished with her yet, and even louder noises were now called for. The audience drumming on the floor with their feet is the image I had when I read "drum roll". While it's certainly not an everyday occurrence at the concerts I go to, I don't think I'd call it "rare". But times and manners vary; if the ladies at Meiningen were terribly conservative and stuffy by the standards of today (or even of those days, going on Reger's crack-glove comment), then a 30-year gap between occasions of audience drumming might well happen. So, I'm interpreting the Tusch as audience noise. Is this reasonable? Or could it mean that she'd signalled to the conductor that she was coming back to perform an encore, and he decided an appropriate introduction was called for under the circumstances, so he got the drums and trumpets to herald Alma in with a fanfare, drum roll and cymbal clash? He wouldn't have done this on any old occasion (I've never encountered it), so he must have had a special reason to do this. Exceptional levels of audience applause would be such a reason. What I'm trying to get at is: was the Tusch a statement by the audience, or by the orchestra, or both? Or is there not enough information to be sure? -- JackofOz (talk) 20:18, 1 December 2008 (UTC)[reply]
I thought about that too. I have never heard Tusch (remember I'm always talking about the German word used in German, not "Tousch" used in English) used for audience behavior. Nor could I find anything in German music dictionaries or online. The Tusch can trigger audience behavior by signalizing "applause!" to the audience (as in the circus). I'm leaning toward your last interpretation (conductor deciding to give the orchestra a cue for performing a Tusch much to the audience's excitement). ---Sluzzelin talk 21:48, 1 December 2008 (UTC)[reply]
I've never heard of a Tusch being a drum roll... or any percussion performance for that matter. (The circus thing, for me, is just a Trommelwirbel.) Nor have I ever heard about it as coming from an audience; it's not a word I'd use or understand in this context. A Tusch is a fanfare, especially by wind instruments. Have a look at "Tusch" at youtube, e.g., here. --Ibn Battuta (talk) 10:03, 3 December 2008 (UTC) PS: Not quite the same as this one at any rate...[reply]
OK, fanfare it is. Thanks, all. -- JackofOz (talk) 22:50, 5 December 2008 (UTC)[reply]

How do I demonstrate that animals do not have religion

How do I demonstrate that ( schools of )dolphins do not have religion and cannot form religious cults? 122.107.203.230 (talk) 11:22, 1 December 2008 (UTC)[reply]

While not answering your question, it's possible whales do form religious cults. They seem to commit mass suicide an awful lot. Also a lot of cats thing they're God Nil Einne (talk) 12:02, 1 December 2008 (UTC)[reply]
The first step is to define "religion" and "religious cult". --Tango (talk) 12:53, 1 December 2008 (UTC)[reply]
You could ask the IRS if they have any dolphin religions registered[14]. This article[15] discusses the concept of what is a religion in US law. James Madison called religion "the duty which we owe to our creator, and the manner of discharging it", which is rather vague. There is some question in US law of whether religion requires a belief in God, but in 1965 in United States v. Seeger the Supreme Court suggested a religion is "a given belief that is sincere and meaningful [and] occupies a place in the life of its possessor parallel to that filled by the orthodox belief in God". This would seem to suggest both belief and practice are involved (so it's not merely taking your dog to church, your dog must have some beliefs about god, morality, eschatology, transcendence, or similar). Philospher Daniel Dennett thinks animals have beliefs at least in some minimal sense.[16] Therefore it would appear hard to show that animals do not have religion, if they are capable of both belief and religious practice. Of course, the definition under US law is only one definition of religion, and you might say that a dog's relationship with his or her owner is analogous to that of a human being with God. --Maltelauridsbrigge (talk) 14:46, 1 December 2008 (UTC)[reply]
Perhaps one of the most common features associated with religion in humans is elaborate death rituals. That is, instead of simply discarding their dead, a formal burial, cremation, etc., is often performed. The problem is, I don't see how this would be possible for dolphins. Perhaps if they always beached their dead on a particular beach, that might qualify. StuRat (talk) 14:55, 1 December 2008 (UTC)[reply]
That still leaves the problem of definition, unfortunately. After all, non-religious humans also take part in elaborate death rituals. Algebraist 15:34, 1 December 2008 (UTC)[reply]
I'd argue that the elaborate death rituals of religious humans have rubbed off on those who aren't. If no humans were religious, I'd imagine human body disposal would be much simpler and based mainly on hygiene requirements. StuRat (talk) 23:51, 1 December 2008 (UTC)[reply]
Elephants surround and seem to comfort each other as they die.[17] Anyone know about dolphins? Maybe whale mass-beachings?
Whale beachings mainly seem to cause death, not be the result of death. StuRat (talk) 23:51, 1 December 2008 (UTC)[reply]
Yes similar to cult mass suicides. Those that do it are probably rather miffed at the humans who stop them going to whale heaven Nil Einne (talk) 09:35, 2 December 2008 (UTC)[reply]

I have heard miscellaneous birds outside start to twitter in what seemsed a very reverent manner when the sun came up in the morning. It seemed they might be sun worshippers. Edison (talk) 06:19, 2 December 2008 (UTC)[reply]

Presumably to have a religion you have to have a conscious mind (spelling), do other animals have what we could call consciousnous (again spelling)? Without that i doubt they could formulate what I would consider a religion - which would be presumably the believe in a higher-power/that sort of thing. 194.221.133.226 (talk) 15:40, 2 December 2008 (UTC)[reply]

Prophets for profit

Isn't the tax-free status of churches in the US incompatible with the separation of church and state ? That is, if churches enjoy the benefits of taxes, like roads to bring their flocks in to be shorn, without paying their portion for those benefits, doesn't that amount to a state-sanctioned subsidy for the church ? StuRat (talk) 15:02, 1 December 2008 (UTC)[reply]

Have you read Establishment Clause of the First Amendment#Financial assistance? Algebraist 15:09, 1 December 2008 (UTC)[reply]
Walz v. tax commission of the city of New York seems to be the most relevant Supreme Court case. Algebraist 15:15, 1 December 2008 (UTC)[reply]
That seems quite relevant, but also quite long. Do we have a summary ? StuRat (talk) 15:37, 1 December 2008 (UTC)[reply]
There's no Wikipedia article as yet. Google has some summaries, such as this. Algebraist 15:42, 1 December 2008 (UTC)[reply]

Re: the section header -- Due to a settlement document signed by the IRS to end an IRS-Scientology legal battle, Scientology gets special extra tax breaks which no other religion is entitled to (see [18]). AnonMoos (talk) 19:54, 1 December 2008 (UTC)[reply]

The original purpose of separation of church and state was not to stop a certain religion from becoming powerful within national government, but rather to prevent government from becoming influential in matters of religion. Hence, the tax break. Now, to be fair we should also agree that any recognized religious leader or institution that participates in political affairs should lose that tax-free status. DOR (HK) (talk) 05:19, 2 December 2008 (UTC)[reply]

Not sure that's entirely true; Thomas Jefferson, at least, was stronly skeptical of any influence by organized religion on government... AnonMoos (talk) 11:14, 2 December 2008 (UTC)[reply]

The United States Establishment Clause uses the term "establishment" and does not specifically state separation of church and state. Reading quite a few Supreme Court cases, I saw no principled explanation for the differences in accomodationist vs. separtist holdings. The New York Times ran an insightful and disturbing series on this very topic within the past two years.75Janice (talk) 02:16, 3 December 2008 (UTC)75Janice[reply]

Librarian looking to help

How does one get involved as a volunteer on the reference desk?LibraryPix (talk) 15:26, 1 December 2008 (UTC)[reply]

Reply to questions when you've got an answer. That's it, really. The reference desk follows the general wiki principle of collaboration; anyone with something to contribute is free to do so. For more detail, there's Wikipedia:Reference_desk/Guidelines. -- Consumed Crustacean (talk) 15:31, 1 December 2008 (UTC)[reply]
You could also contribute to the rules, by letting us know what a real Reference Desk (in a library) does in the following situations:
1) When asked to give a medical diagnosis, legal opinion, or other professional advice.
2) When asked a matter of opinion.
3) When asked something you know the answer to personally but can't find a source to verify (AKA, "original research"). StuRat (talk) 15:43, 1 December 2008 (UTC)[reply]
Welcome to the Ref Desk. Whenever I've asked a Real Life Reference Librarian for help responding to a question here, I have been very favorably impressed. Here we often just go with references which are available free online, You have acess to lots more online databases, such as Proquest and other premium databases. You also have many expensive reference books readily available. I look forward to your input. Edison (talk) 21:13, 1 December 2008 (UTC)[reply]

To permanately end terrorism by Extremists

What would need to happen in order to end terrorism by Muslim extremists from the Middle East and Asia, short of completely annihilating the USA? What exactly are they after?--Emyn ned (talk) 16:03, 1 December 2008 (UTC)[reply]

Killing the enemy will not end violence. A new enemy will be chosen. The only way to end war of any kind is to kill all humans. Then, we only need to be concerned with minor scuffles between animals and insects - especially ants. Those little buggers really get into violent encounters. Honestly, if you want to have a rather enjoyable read on this topic, try TH White's The Once and Future King. Much of the book is spent trying to figure out what to do with an army (or a few knights) when they aren't needed for combat anymore. -- kainaw 16:07, 1 December 2008 (UTC)[reply]

I wasn't trying to say that they should be killed. I was thinking, if we (USA, Europe) gave them what they want or met their demands, what would they be? Also, how will it change our world? --Emyn ned (talk) 16:10, 1 December 2008 (UTC)[reply]

Power? The thrill of violence? The religious reasons given are generally just excuses, as is made obvious by the fact that the vast majority of Muslims don't support terrorism. --Tango (talk) 16:46, 1 December 2008 (UTC)[reply]
You are placing logic in a completely illogical situation. There are no demands that must be met. If anyone were to meet any demands, there would be new demands. If everyone we consider a terrorist were killed, there would be new terrorists. Humans fight just to fight. There is no logic. Just look at the justifications for fighting - they never make sense. So, as I said, the only way to stop the fighting is to kill all humans (not just the terrorists, all humans). -- kainaw 16:46, 1 December 2008 (UTC)[reply]
That's an overly simplistic explanation, if not completely wrong. As they say, war is a continuation of politics by other means and terrorism is no different. Virtually all terrorists groups have political goals. Whether it's the PLO trying to establish a Palestinian state or the IRA trying to win independence from Great Britain or Christian terrorists trying to stop abortion, it's all about politics. 216.239.234.196 (talk) 18:04, 1 December 2008 (UTC)[reply]

Ok, I see your point. Thanks for answering. --Emyn ned (talk) 16:57, 1 December 2008 (UTC)[reply]

Well, first of all, Islamic terrorists aren't a homogeneous population. Different groups have different agendas, some of which are contradictory. Just look at Iraq where you have Sunni terrorists fighting Shiite terrorists. 216.239.234.196 (talk) 18:04, 1 December 2008 (UTC)[reply]
The problem with attempting to claim that the members of a terrorist group (or any group that participates in violence) have an agenda is that the members usually do not have an agenda. They are just there to fight. If the leaders with their agenda go away, the fighters will join some other group so they can keep fighting. Due to an unfounded belief that humans are somehow special, people have always tried to claim that war (of any kind) is caused by something that can be fixed. The simpler explanation is that war is a common result of humans being alive. Not all humans participate in war, but humans naturally divide themselves into groups and humans naturally see the world as a zero-sum game. So, the groups naturally fight. Try to find a single year in the history of humans in which there wasn't combat going on somewhere in the world. It hasn't ever happened. There are always people looking to fight and always somewhere for them to fight. -- kainaw 19:01, 1 December 2008 (UTC)[reply]
Oh, I don't think you can get a suicide bomber to strap a bomb to his chest unless he believes that he was dying for a greater cause. 216.239.234.196 (talk) 19:24, 1 December 2008 (UTC)[reply]
They do it "for the greater glory of Allah" or similar, not for some real world cause - it's simple religious indoctrination. --Tango (talk) 19:31, 1 December 2008 (UTC)[reply]
Religion is a factor, but it really isn't that simple. There are other motives. A good movie to watch about this is Paradise Now. Wrad (talk) 20:03, 1 December 2008 (UTC)[reply]
Kainaw, you can't seriously believe that things can never improve from where they are now. Things are much better right now in some parts of the world than in others. If we could just reduce the rate of terrorism (and hunger, and disease...) in the rest of the world to the rate in, say, western Europe, it would be an enormous improvement. There's evidently no barrier to that in human nature. Educate people, raise their standard of living, and they'll stop killing each other. The evidence is everywhere. And what's this about "an unfounded belief that humans are somehow special"? Humans are obviously special, among extant species, in all kinds of ways. You seem to be saying that we'll never be rid of war because humans are just like other animals, but not many animal species wage war in the first place. (Single combat, sure, but that's not at all what we're talking about here.) -- BenRG (talk) 22:17, 1 December 2008 (UTC)[reply]
A lot of terrorists are from western Europe. They often go to training camps in Pakistan, and similar, but they are frequently citizens of the countries they are bombing. --Tango (talk) 22:41, 1 December 2008 (UTC)[reply]

The answer to your question is opportunity. Terrorism, as it is referred to now, cannot end. It's been around since before the establishment of the USA. But, if opportunity is spread, the numbers of volunteers will fall. This happened in Northern Ireland and I think it serves as a good example (albeit a small one) of conflict resolution. There are still a small number of Irish Republicans (estimated at around 100) still targeting police officers and the Loyalists aren't very active either. The large majority of the general public (>90%) refuse to support any form of political violence there. This was not the case twenty years ago. Removing volunteers requires the removal of public support, which is small in the Islamic world but high in certain areas where the terrorists are strongest. Support can be removed by creating opportunity through investment and non-partisan media controlling the airways. How this can be done is anybody's guess. Donek (talk) 20:04, 1 December 2008 (UTC)[reply]

Yes, terrorists exist because there is nothing else to do that provides the same acceptance. Many of them come from countries or areas where there are few economic opportunities and where terrorists are glorified by the public. People want to be accepted, if you have no job and your life seems to have no future, you will obviously be more easily persuaded to blow yourself up for some cause. Wrad (talk) 20:19, 1 December 2008 (UTC)[reply]

Step 1: Stop classifying all terrorists as Muslims and all Muslims as terrorists (cf Timothy McVeigh, Red Brigades, Red Army Faction and Japanese Red Army). Step 2: Stop classifying the problem as one of “Muslim extremists from the Middle East and Asia” vs. the USA. (cf November 2008 Mumbai attacks) Step 3: Stop assuming that there is some unified organization that will define objectives and negotiate a settlement, as should be obvious from the previous two examples. Step 4: Reread kainaw‘s post about (il)logic. DOR (HK) (talk) 05:31, 2 December 2008 (UTC)[reply]

To end hunger all over the world..

Hypothetically, say the USA alone decided that it will end world hunger right now, all by itself and it had unlimited funding (also pretend that the worldwide credit/mortgage crisis didn't happen). What would USA do first? How will it attack this? What will need to happen? And, after every impoverished country has the food, resources and knowledge to feed itself, how will this benefit the USA in the short and long run? --Emyn ned (talk) 16:15, 1 December 2008 (UTC)[reply]

By your last two questions, it appears that you have mistaken this reference desk as a discussion forum. This is not a forum to discuss topics. This is a place to get references. If you want a reference on a specific topic, please ask. If you want to discuss hypothetical situations, please use one of the millions of discussion forums available on the internet. -- kainaw 16:48, 1 December 2008 (UTC)[reply]

I am not here to "discuss" this. I want to know exactly what has to happen in order to end world hunger. Assuming my teacher is right, we have the resources (USA, Europe) to do this. But when I asked him, "OK, How?", he had no answer. Aside from politcal and other hurdles, what is the first thing to do? --Emyn ned (talk) 16:55, 1 December 2008 (UTC)[reply]

As Tango says below, it is a problem with warlords, terrorists, corrupt governments, etc... It is not a problem with food production. How do you get rid of all warlords, terrorists, and corrupt politicians? Kill all humans. If there are just two humans left, at least one will try to steal from the other (but I suspect they'll steal from each other). -- kainaw 17:05, 1 December 2008 (UTC)[reply]
There is plenty of food in the world, the problem is getting it to where it is needed, which is a political problem. There is no obvious solution - invading the whole world and putting it under a totalitarian regime that ensures food gets to the right places would work, but I doubt it's a very popular solution. A less drastic form of world government might also work, but it's rather difficult to implement such a plan. --Tango (talk) 16:50, 1 December 2008 (UTC)[reply]

Ok, so all political issues aside worldwide, hypothetically. If there were no poltical issues whatsoever (which I know is impossible), what would be the first logical step in order to end world hunger? Ship extra food to the impoverished countries or send seeds and farm equipment? And once this is all done, I wonder how the world would change specifically? --Emyn ned (talk) 18:34, 1 December 2008 (UTC)[reply]

If there were no political problems, there would be no mass hunger. The free market would probably do the job just fine (there may be a need for food aid sent to some of the absolute poorest, but most would be able to buy food if it weren't for political problems in their country). --Tango (talk) 19:25, 1 December 2008 (UTC)[reply]
That seems quite optimistic. While it is certainly true that food shortages are often connected with political corruption or unwise government policies, there are many other factors, economic ones being at the fore, and I wouldn't rely on the free market to address these. Warofdreams talk 00:53, 2 December 2008 (UTC)[reply]
If there was one peaceful world gov, the next step after distributing food would be to distribute birth control and/or embark on a massive sterilization program to prevent the third world population, no longer limited by starvation, disease, and war, from increasing to the point where even the world gov couldn't feed them any more. StuRat (talk) 19:55, 1 December 2008 (UTC)[reply]

Let us start with a really simple one before we get to the more complex global problems:
Hypothetically, say the USA alone decided that it will end injustice in the Iraq right now, all by itself and it had unlimited funding. What would USA do first? How will it attack this? What will need to happen? And, after this dictatorial country has the food, resources and knowledge to free itself, how will this benefit the USA in the short and long run?
Having solved this in a jiffy, we (or rather, you, God´s own country) may now proceed to problem 2: world hunger. --Cookatoo.ergo.ZooM (talk) 20:05, 1 December 2008 (UTC)[reply]

It is not just political, there are physical constraints which are putting up food prices beyond the reach of the poor. Modern agriculture depends on the energy from cheap oil and also a water supply, which is another problem. Farming the moon under an artificial environment, using a solar powered electromagnetic railgun to transfer the food to Earth and parachuting the food to ground where it is required would be one solution. The water could be obtained from comets. Once nuclear fusion works, massive quantities of seawater could be desalinated and return desert areas to agriculture, though this could be done with solar energy. But with fusion multi-story underground buildings with solar lamps could supply indefinite amounts of food, given virtually free energy. Later, replicator nanotechnology could produce food efficiently from dirt-but energy is the bottom line in food production, because indirectly we all rely on pure energy from the sun-until a sun can be created on earth. One other method would be to redirect rivers to desert areas across national boundaries, but whilst it might be politically acceptable for Canada to divert some of its massive water resources to the United States, It is less likely that Europe or Russia would do this for hostile Islamic states on its Southern border, though a big enough food crisis might make this situation viable.Trevor Loughlin (talk) 04:19, 2 December 2008 (UTC)[reply]

Tango laid out the key to rephrasing the question. The next part is “what are the consequences of moving surplus food to places of scarcity?” The first one that comes to mind is the economic impact: where food is scarce, farmers earn more; add a significant amount of food to that economy, and farm incomes would go down. Next question: is that a desirable outcome, or would impoverishing farmers simply make matters worse? (When you get the answer, Emyn ned, call the World Bank and offer your services as their new boss.) DOR (HK) (talk) 05:39, 2 December 2008 (UTC)[reply]


Will do. There is nothing wrong with dreaming big! Sometimes, miracles happen. --Emyn ned (talk) 14:15, 2 December 2008 (UTC)[reply]

it amazes me that no one has talked about food tariffs - the US and the EU both impose massive tariffs from imported food so the third world has very little incentive to produce food to sell (this problem is conpounded by the fact that they then dump tonnes of cheap, subisdised food on to the developing markets). Get rid of these and you'd solve a great deal of food shortage problems, and save the tax payers of these countries a pretty penny too...82.22.4.63 (talk) 16:38, 2 December 2008 (UTC)[reply]

(This isn't too proper an RD/H question, but ..) 82.22.4.63 has a very good point, falling into the category of "stop doing many things that make world hunger worse." Ethanol policies are another one. US military activities, promoting arms races, destabilizing and invading countries and "defense" spending is another. The operations, e.g. Structural adjustment of the World Bank and the IMF of the last few decades could hardly be designed better to promote world hunger and prevent development in poor countries. The lot of the world's poorer countries would be helped by these "international loan-sharking institutions" (the late Jonathan Kwitny's felicitous phrase) not existing; of course if they were actually used to promote Marshall plan style development - dream on, that would be great. One of the great long-term benefits of the Bush years has been the weakening of these institutions.John Z (talk) 23:13, 2 December 2008 (UTC)[reply]

Read the book "God's Men" by Pearl S. Buck. It has all answers to the world's hunger problem. As for the benefit part, the premise is that people would favour a government who provides them food (the book was published in 1951, so this might no longer be true). As for the part about benefitting USA, the book attempts to answer that too. Jay (talk) 13:08, 3 December 2008 (UTC)[reply]

E.coli

Can anyone come up with a good estimate for the worldwide market size for diagnostic products for E. coli detection? Donek (talk) 19:49, 1 December 2008 (UTC)[reply]

That would depend entirely on the product and cost. If you had a one cent strip you could stick in any suspect food, and the color would tell you instantly if it's infected with E. coli, then you could sell maybe 100 billion such strips a year. If you're talking about a $1,000,000 E. coli diagnostic machine, then there may not be any market for it at all. StuRat (talk) 20:09, 1 December 2008 (UTC)[reply]

I don't think the strip you refer to exists but I take your point. It's pretty cheap and easy to use. I'm not looking for possible sales of a particular product, just the current market for cheap diagnostic products. Donek (talk) 20:22, 1 December 2008 (UTC)[reply]

Same question, same answer. Define "cheap." Are you thinking of "Cheap to the average Swiss banker," or "Cheap to the average Bangladeshi farmer" ?DOR (HK) (talk) 05:41, 2 December 2008 (UTC)[reply]

Question withdrawn. Donek (talk) 19:25, 2 December 2008 (UTC)[reply]

Wait Wait Wait!
Sites like [19] have the type of market research you're looking for. [20] [21] Here are some other reports, that focus on test strips in medical settings. They're pricey, but may be worth it for a serious investigation. [22] [23] Here are some examples of similar products.
Ah. [24] here is a rough estimate of the size. $1.67 billion by 2010 with 55% in the US and Europe. The "pathogen testing" sub-market represents $992 million. This article has plenty more report links to the exact market reports you're looking for.
If you're putting together a business plan, financiers understand that different products classes and types are going to have different price elasticities. They also understand that inserting a unique or new product into a market will typically increase the size of that market. Otherwise, when someone asks for "market size" they usually just mean "revenue per year currently allocated to addressing that want/need", regardless of product size, type, price, etc.NByz (talk) 19:32, 2 December 2008 (UTC)[reply]

Thanks for the resource. It's a project we're doing for uni. I withdrew the question because we changed the organism to MRSA which has a market of about US$50,000,000. I should've explained why I removed the question but thanks for trying anyway.

There have been quite a few cases of that in one of our hospitals here in victoria, BC.NByz (talk) 21:40, 2 December 2008 (UTC)[reply]

Chadian City

Apparently there's a Chadian city named Saigon. However, the only reference I can find about this locale is on maps. Does this place exist in real life or only in the mapmakers' imagination? DHN (talk) 20:03, 1 December 2008 (UTC)[reply]

I would suspect there are a LOT Of places in Chad which you can't find anything about on the internet. The fact that it has the name Saigon may seem to make people more likely to talk about it but then again, there's no Saigon anywhere else now so it's not so interesting as it may have been Nil Einne (talk) 20:32, 1 December 2008 (UTC)[reply]
N.B. Some things say Saigon, Biltine. If they mean Biltine Prefecture that doesn't exist anymore. So who knows if Saigon in Chad still exists either? It's possible they mean Biltine Department but that's bit odd since you'd normal specify the region I presume. Nil Einne (talk) 20:35, 1 December 2008 (UTC)[reply]
Judging from the apparent irrigation canals there in the satellite view [25], I'd say it's a small farming community. And, judging by the name, I'd guess it was settled by Vietnamese fleeing the Vietnam War. StuRat (talk) 20:29, 1 December 2008 (UTC)[reply]
The official languages of Chad are French and Arabic. It used to be part of French Equatorial Africa. And the name "Saigon", with that spelling, was established for the Vietnamese city now known as Ho Chi Minh City by the French in 1862 (according to the Ho Chi Minh City page). So perhaps the Saigon in Chad was named during the French colonial era as well, perhaps after the capital of colonial French South Vietnam (Cochinchina). Then again, "Saigon" may be transcription from an Arabic spelling that just happens to have been rendered as "Saigon". This was my first thought after reading the question. Or perhaps it was founded by Vietnamese people, as StuRat said. Fleeing Vietnam for Chad seems an odd idea to me, but I know little about where Vietnamese people have fled to, or why. Maybe the French connection between Vietnam and Chad made it sensible for Vietnamese to flee to Chad.
But to respond to the original question, there doesn't seem to be much on the web about Saigon, Chad, although a search for it at NGA GNS works. I would guess that Google got its placename data for places like Chad in part from the NGA GNS database. I can't say whether it exists "in real life", but the US government seems to think so, whatever that is worth. Pfly (talk) 07:15, 2 December 2008 (UTC)[reply]
Standard Arabic doesn't even have a "g" sound (though it exists in some dialects, most notably Egyptian). AnonMoos (talk) 10:57, 2 December 2008 (UTC)[reply]
There are many languages spoken in Chad other than French and Arabic. According to Ethnologue, for example, the language of the Maba people had 250,000 speakers in Biltine Prefecture as of 1999. It seems quite likely that this is a local place name that is coincidentally the same as the former Vietnamese place name. The phonemes in the name are quite common ones among world languages. Marco polo (talk) 02:00, 3 December 2008 (UTC)[reply]

Article translation database

Is there a database of translations of articles? (Like "Index Translationum" does for books.)66.245.141.147 (talk) 20:27, 1 December 2008 (UTC)[reply]

What kind of articles do you mean? Wikipedia articles? There is a list of version in other languages on the left hand side of every article. --Tango (talk) 21:13, 1 December 2008 (UTC)[reply]
Versions as in not direct or cross-translations – you might find some are more brief, less developed or more developed than the English equivalent. Still useful though, Julia Rossi (talk) 22:14, 1 December 2008 (UTC)[reply]

Resources on Gorky and God Building

Are there any good books/websites which give infomration about Gorky and Godbuilding? I have read that embalming Lenin was part of an attempt to resurrect him and others at some future date. Particularly in what Zizek calls mystical Marxist pantheism. ""bio-cosmism," the strange combination of vulgar materialism and Gnostic spirituality which formed occult shadow-ideology, the obscene secret teaching, of the Soviet Marxism. Repressed out of the public sight in the central period of the Soviet state, bio-cosmism was openly propagated only in the first and in the last two decades of the Soviet rule; its main theses are: the goals of religion (collective paradise, overcoming of all suffering, full individual immortality, resurrection of the dead, victory over time and death, conquest of space far beyond the solar system) can be realized in terrestrial life through the development of modern science and technology. In the future, not only will sexual difference be abolished, with the rise of chaste post-humans reproducing themselves through direct bio-technical reproduction; it will also be possible to resurrect all the dead of the past (establishing their biological formula through their remains and then re-engendering them - at that time, DNA was not yet known...), thus even erasing all past injustices, "undoing" past suffering and destruction. In this bright bio-political Communist future, not only humans, but also animals, all living being, will participate in a directly collectivized Reason of the cosmos... Whatever one can hold against Lenin's ruthless critique of Maxim Gorky's the "construction of God (bogograditelk'stvo)," the direct deification of man, one should bear in mind that Gorky himself collaborated with bio-cosmists. It is interesting to note resemblances between this "bio-cosmism" and today's techno-gnosis. - " --Gary123 (talk) 23:14, 1 December 2008 (UTC)[reply]

Excuse me? Emma Dashwood (talk) 07:01, 2 December 2008 (UTC)[reply]
@ Maxim Gorky there's this: Most controversially, he articulated, along with a few other maverick Bolsheviks, a philosophy he called "God-Building", which sought to recapture the power of myth for the revolution and to create a religious atheism that placed collective humanity where God had been and was imbued with passion, wonderment, moral certainty, and the promise of deliverance from evil, suffering, and even death. Though 'God-Building' was suppressed by Lenin, Gorky retained his belief that "culture"—the moral and spiritual awareness of the value and potential of the human self—would be more critical to the revolution’s success than political or economic arrangements. You might find links at the end of the article. Apart from the subject, and Slavoj Žižek's input, is there a question beyond the first sentence? Julia Rossi (talk) 08:20, 2 December 2008 (UTC)[reply]

Poster in the West Wing (TV serial)

In the TV serial The West Wing, the character Toby Ziegler has a poster in his office (at least the first season) which looks lika a reproduction of an art print [26]. Does somebody know who the artist is? I have posted this question in the Entertainment Reference desk, but nobody knew the answer. /B****n (talk) 23:38, 1 December 2008 (UTC)[reply]

You could try and find out who the art director is (or for that matter anybody in the art department stands a chance of knowing) and trying to contact them by looking for their CVs or contact information on Google. It does depend on how strongly you want to find out - and you might not be able to get anybody's contact details. Another option is to go to the newsgroups and forums of the show and ask there - just in case someone noticed the picture and knows who did it. Rfwoolf (talk) 18:00, 2 December 2008 (UTC)[reply]

Anglo-Boer War diaries

Question moved from the midst of the hunger question and given its own heading. Gwinva (talk) 03:16, 2 December 2008 (UTC)[reply]

Did Francis Jeune who was the Judge Advocate General during the second Anglo-Boer War, or his deputy, left/wrote a bookor a diary about it? Did Sir Robert Finaly or/and Sir Edward Carson who were both the Solicitor General of the Crown and part of the Law Officers of the Crown (gave legal advises to the Cabinet and to the government)at the time of the Second Anglo-Boer War, wrote books or left diary on that time?

Oded89.138.121.145 (talk) 17:51, 1 December 2008 (UTC)[reply]

Indigenous Australian population

Is there a site where I could find in-depth statistics about Indigenous Australian population from 1788 to present?— Preceding unsigned comment added by 220.244.107.151 (talkcontribs)

Strangely enough, we have LOTS of articles on Indigenous Australians. That's probably a good launching point. If you dig deep from there, and follow the sub articles and/or references from that maun article, you will likely find what you are looking for. --Jayron32.talk.contribs 04:30, 2 December 2008 (UTC)[reply]
There are presumably few reliable quasi-exact statistics available until after they had already been seriously impacted by Europeans, however... AnonMoos (talk) 10:53, 2 December 2008 (UTC)[reply]

Active yoga cultures ?

I added a suitable title. StuRat (talk) 05:13, 2 December 2008 (UTC)[reply]

1)Spritual Dimension of YOGA?

2)Yogic concept of human body?

3)Mental health for better living through Yoga?

4)Application of yoga in Education?GEENA SAJITH (talk) 04:37, 2 December 2008 (UTC)[reply]

Is any of this answered in our yoga article ? StuRat (talk) 05:15, 2 December 2008 (UTC)[reply]

Is Yoga is a science and art ?

1.Yoga is a Science & Art ?

2.Role of Yoga in the New Millennium?

Let me google that for you like this and like this manya (talk) 09:12, 2 December 2008 (UTC)[reply]

British Ministers

These days, British ministers (not only cabinet-level, I'm referring generally also to Ministers of State and Under-Secretaries of State) are generally MPs. In the (rather distant) past they have often been Peers (okay, Members of the House of Lords, but we have to go quite far back to see the last Lord spiritual serving as a Minister). A couple of Lords (temporal) are still in Cabinet, most notably Lord Malloch Brown and Lord Mandelson. Question: Is there a statutory or other "laid down" requirement that ministers be either MPs or HoL members? I reckon, obviously, that it is an asset for Question Time to be a member of either house, but you could say that as long as one minister in the department is a member of either house (read: one Under-Sec for the HoL and one Under-Sec for the HoC), that would suffice and the Secretary of State needn't be a member of either house. Thoughts on this? (Yes, Cabinet is technically a Committee of the Privy Council, so cabinet ministers are Privy Councellors but that doesn't have anything to do with membership of either house of parliament). If this question is completely confusing, I'll try to rephrase... --Mbimmler (talk) 12:14, 2 December 2008 (UTC)[reply]

It is my understanding that it is not necessary for Ministers to be a member of either house, although it has long been thought desirable to find a seat for any non-Parliamentarians who are to be brought in to a cabinet. For example, Malcolm MacDonald lost his seat at the 1935 general election, but was appointed Secretary of State for Dominion Affairs regardless, and did not return to Parliament until he won a by-election in February 1936. A parallel example: David Bleakley was appointed Minister for Community Relations in Northern Ireland despite not being a member of either house there (he was legally limited to serving for only six months, but I don't believe that there is a similar restriction for the Westminster Parliament). Warofdreams talk 13:18, 2 December 2008 (UTC)[reply]

Hello! Can anyone let me know a good link where i may found a full report on the novel'Gone with the wind'by Margret Mitchell? Cheers.

Check the sources in our articles on Margaret Mitchell and Gone With the Wind. You can probably write a very good one yourself by reading up on the various sources described. In fact, my guess is, your teacher wants you to. :-) (A full report - makes it sound like you want the result of an investigation. :-)
The library will assuredly have most othe the books cited as references (I sure hope our articles have references, now that I say that. :-) ) But, remember that what you read should be part of it, too, as this will show your ability to grasp and evaluate the information you read.
(Voice of worried parent) You...er..did read it, didn't you?Somebody or his brother (talk) 14:33, 2 December 2008 (UTC)[reply]
Note: If you merely watch the movie, there is material there not found in the novel, and considerable material from the novel omitted from the movie. Edison (talk) 15:00, 2 December 2008 (UTC)[reply]

Bangladesh vs. Italy and Greece

Why Bangladeshi people migrate to these countries? Is it because they have a history with each other since the birth of Bangladesh?

If you're talking about illegal immigration, probably because those countries are easier to get to without expensive airline tickets. The country in Europe with which Bangladesh has the strongest historical ties is the former colonial power, Britain. AnonMoos (talk) 16:11, 2 December 2008 (UTC)[reply]
If you're talking about legal immigration, the odds are that two factors play an overwhelming role: the difference in standards of living and the degree of difficulty involved in obtaining the right of abode. DOR (HK) (talk) 08:21, 3 December 2008 (UTC)[reply]
I don't understand why there would be significant legal Bangladeshi immigration to Greece, considering that the countries have very few meaningful historical or cultural ties, and Greeks and the Greek government typically do not see themselves as any kind of country of immigration (so that Greece is currently in trouble with the EU for seemingly not even seriously considering applications for political asylum), and Greeks generally strongly do NOT want significant additional immigration of Muslims... AnonMoos (talk) 08:31, 3 December 2008 (UTC)[reply]
Bangladeshis in Britain are mainly from Sylhet and the links are very long-standing. Most "Indian" restaurants in Britain are actually run by Sylhetis. I wonder whether some Bangladeshis in Italy and Greece are there in order to start up restaurants for tourists? I met someone in Portugal who was doing just that, but then it was in a town much frequented by the British. Itsmejudith (talk) 21:48, 3 December 2008 (UTC)[reply]
AnonMoos, perhaps the determining factor is along the lines of what Bangladeshi migrants want -- a European residence -- and less about what Greeks want. DOR (HK) (talk) 07:21, 4 December 2008 (UTC)[reply]

Peter Mandelson - Inner Steel

Does anyone know where I could find a video recording of Peter Mandelson delivering his surprising and incredible "inner steel" speech in Hartlepool during the 2001 general election? I've been looking on youtube but their politics clips are disappointing. Richard Hock (talk) 16:14, 2 December 2008 (UTC)[reply]

Do you know of any examples where a political party's logo has the same colours or makes strong usage of that same nation's national flag?

Also, do you know if this is disallowed anywhere?

The reason I ask is because flags have strong significance in propaganda, and a political party that uses the nation's flag may be unfair - in fact if it were allowed (and I think it is) I would have assumed most political parties would want the flag in their logo.

My own example is the logo for COPE which uses the colours of the South African flag.

Rfwoolf (talk) 17:22, 2 December 2008 (UTC)[reply]

The logo of the British National Party consists of the letters BNP overlaid on a Union Jack. Fortunately (POV alert) the BNP is very much a minority party. AndrewWTaylor (talk) 17:28, 2 December 2008 (UTC)[reply]
The Official Monster Raving Loony Party's logo also uses that Union Jack overlaid by the party name, in this case superimposed on a shield, below a top hat. Warofdreams talk 18:04, 2 December 2008 (UTC)[reply]

See Sinn Fein, DUP and UUP. Maybe Hamas as well. Donek (talk) 19:31, 2 December 2008 (UTC)[reply]

The Liberal Party of Australia's logo incorporates the entire Flag of Australia. but leaning to the right, as you'd expect. The Australian Labor Party uses a surprisingly right-leaning Southern Cross, the Federation Star, and red and blue (different shades from those in the flag) - but not the Union Jack. The National Party of Australia's logo uses green and gold, the National colours of Australia. -- JackofOz (talk) 20:25, 2 December 2008 (UTC)[reply]

Both major parties in the United States use Red White and Blue in their logo. See Democratic Party (United States) and Republican Party (United States). ---Sluzzelin talk 20:52, 2 December 2008 (UTC)[reply]

The Liberal Party of Canada is red and white and also has the maple leaf (though not, I suppose, the entire actual flag) in the logo. Adam Bishop (talk) 01:55, 3 December 2008 (UTC)[reply]

I think its very common, throughout the world, that political parties use national colours. In India, the national colours are used by the Indian National Congress and its off-shoots. However, there are some exceptions. In France, I think parties are banned from using the tricolor in elections (I heard this somewhere, any reference?). --Soman (talk) 07:33, 3 December 2008 (UTC)[reply]

How about a reverse case? The Nationalist Party (Kuomintang or KMT) flag is the top left corner of The Republic of China (i.e., Taiwan) flag. DOR (HK) (talk) 08:27, 3 December 2008 (UTC)[reply]

How about a flag that's the same for the party and the country: Nazis. BTW, "a political party that uses the nation's flag may be unfair" is a cultural assumption--namely that people like seeing their flag all around. Especially in Germany, you almost saw the reverse pattern for years. And also elsewhere: Parties of the political right may be likely to use national colors, those of the political left more likely to use the color red. --Ibn Battuta (talk) 09:32, 3 December 2008 (UTC)[reply]

And don't forget the Front Deutscher Äpfel ! ---Sluzzelin talk 15:05, 3 December 2008 (UTC)[reply]

How to find the translations of a book

Starting with an original published work (with an ISBN), how can I find the translation of it? Is there any general archive that links original works with their translations? Mr.K. (talk) 17:30, 2 December 2008 (UTC)[reply]

Some catalogues, like Copac, also give translations as search results for an original text. As a worked example, Terry Pratchett's Soul Music (paperback ISBN 0-552-14029-5) has a Copac record here. Wheras searching the entire database for all books named "Soul Music" by "Terry Pratchett" gives me all the translations that are registered with this database here. One thing to remember is that even though the work may have been translated, the writer of the original text is still the de facto author. Your mileage may vary considerably though with other authors and catalogues. Special:BookSources is a nice place to start if you're searching for a catalogue to try this with. Nanonic (talk) 17:50, 2 December 2008 (UTC)[reply]
Particularly for books originally written in English, try "Basic Search" the author's name in the U.S. Library of Congress online catalog. Or contact the publisher of the original, particularly as they or the author are likely to hold the translation rights. -- Deborahjay (talk) 01:00, 3 December 2008 (UTC)[reply]

is it okay to cite Noam Chomsky on a political article?

I statement I made first got a [dubiousdiscuss] tag added and then was removed. But it was not at all controversial, it is summarized explicitly by Chomsky and he gives a very good reference. Can I include his firm summary as a reference? He is the most-cited person alive. -- 20:23, 2 December 2008 82.120.107.213

Actually check out this link. "I wrote about it several times, but the editors understand that dissident material, no matter how well documented, doesn’t exist." So my question about Wikipedia is, let's use an analogy, if we're living in Orwell's 1984 and I'm editing the article on Oceana, is it okay to include well-cited references that you can check for yourself on the Internet showing clearly that we were not always at war with Oceana, when the article (and all serious publications, such as the New York TImes) say "we have always been at war with Oceana"? What is Wikipedia's position in these cases? -- 20:29, 2 December 2008 82.120.107.213
Content matters should be discussed on the talk page of the relevant article. This is not a question for the Ref Desk. ៛ Bielle (talk) 21:16, 2 December 2008 (UTC)[reply]
Sorry, it is a relevant matter for the reference desk because I'm not asking about one article, but something affecting hundreds. My simple question is, is well-referenced but suppressed information, for example well-cited by Chomsky, allowed in Wikipedia or would it violate neutrality, original research, etc, etc? -- 21:28, 2 December 2008 82.120.107.213
By the way, it's "Oceania", and the actual phrase in the novel is "always at war with Eastasia". And a lot of the tabulated citations to Chomsky are actually to his linguistic works... AnonMoos (talk) 21:25, 2 December 2008 (UTC)[reply]
And why are you linking to a Norman Finkelstein site?? If anybody is a reliable source in political matters, Norman Finkelstein ain't! AnonMoos (talk) 21:38, 2 December 2008 (UTC)[reply]

If something is "well referenced" then, by definition, it is not suppressed, regardless of the topic. ៛ Bielle (talk) 21:41, 2 December 2008 (UTC)[reply]
I have not looked at the references in question, but there is a LOT to be said about how you word your references. If you present a statement as a broadly accepted fact, where it isn't, that can be problematic. For example, if only Noam Chomsky proposes that "A is true" where as most other sources say "A is false" then you need to report "While many sources generally agree that A is false[1] [2] [3], Noam Chomsky belives that A is true[1]." You also need to be careful, and only use Chomsky's own sources for areas where he is considered a notable commentator. For example, Barack Obama has recently come out in support of a Division IA College Football playoff tournament; however he's not a notable college football commentator so, though he is a notable person in his own right, his opinions on college football shouldn't mean squat and should not be cited in our article on the subject. Just because Chomsky said it doesn't mean it needs to be reported, if he is not considered a notable commentator in that field... --Jayron32.talk.contribs 21:52, 2 December 2008 (UTC)[reply]
This really isn't the place for such questions, which should be at WP:RS/N or the article talk page. Chomsky, or Finkelstein could be reliable sources or usable / notable sources depending on the context. Well-cited is ambiguous - it could mean the number of citations in the piece or the number of times the piece has been cited. Jayron32's advice above is good. You really need to give more information about the particular article and edit to get answers other than sometimes yes / sometimes no. Since Chomsky here is merely summarizing a "very good reference", there probably shouldn't be much of a problem if you use his summary , check the ref and cite both.John Z (talk) 22:48, 2 December 2008 (UTC)[reply]

All right, let me get specific. I want for our article International_law_and_the_Arab-Israeli_conflict to have a lead paragraph. I happen to have a specific summary from Chomsky:


But when I added this information from the lead paragraph of the article I linked (which currently has NO lead paragraph, just a one line saying "this article is about..."), it got deleted. So my quesstion is, can I use this as a reference, since "there is near unanimity" according to Chomsky, who is the most-cited living person?

Chomsky, with his polarized stance on this topic, strikes me as too POV a source to lead the page. The passage fails rhetorically to qualify the scope of this position's support base: a mainstream reader encountering that opening phrase ("There is near unanimity") that goes on to cite a single spokesman for the "consensus" (of whom?), may recall the old toothpaste advertisement, "Four out of five dentists agree..." I suggest a more comprehensive introduction to the issue, perhaps continuing with a trimmed and **adapted** version of your opening lines:
-- Deborahjay (talk) 01:20, 3 December 2008 (UTC)[reply]

Why does Alaska have expensive gas compared to other states?

Wouldn't high oil production in that state make gas cheap?

Raw crude needs to be processed in order to be used by vehicles. So no, producing raw crude need not entail cheap gas prices.--droptone (talk) 20:50, 2 December 2008 (UTC)[reply]
Most Alaskan oil is shipped or piped south. Much is refined in California. You can see our list of oil refineries that Alaska does have some refining capacity, but I'm pretty sure that it's all just to help standardize some of the heavier crude for shipment (so it doesn't stick in the pipes, for example). Most gasoline production in the US takes place in our around the Gulf of Mexico. Some in California.NByz (talk) 21:38, 2 December 2008 (UTC)[reply]

December 3

Image of branches in Jewish monotheism.

Does anybody have a link to an image that shows Judaism with offshoots? I'd like to see one with major and minor branches. Compare:


Well, the main outline would be that in the Maccabean / early Roman period there were many "sects" (if you want to call them that), such as Pharisees, Sadducees, Zealots, Essenes, Therapeutae, etc., but the only two that survived in the long run after the destruction of the Temple in 70 A.D. were the Samaritans (already quite separate even before the Maccabean period) and Rabbinic Judaism. Those Jews who did not ultimately affiliate themselves with Rabbinic Judaism after 70 A.D. ended up converting to Christianity in large numbers (such as a significant part of the Jewish community of Alexandria). In the medieval period, the main division was between Jews who accepted the Talmud, Qaraites (who consciously rejected the Talmud), and certain remote Jewish communities (such that in Abyssinia) who were too isolated to be significantly influenced by the Talmud. In the early modern period, there was the division between Chasidim and Mitnagdim. The modern divisions between Orthodox, Reform, Conservative didn't really get established until the 19th century... AnonMoos (talk) 07:10, 3 December 2008 (UTC)[reply]

I've not seen an image, but Q: What's Jewish polytheism? --Dweller (talk) 14:48, 3 December 2008 (UTC)[reply]
Asherah and Ashtoreth, I guess... See Bible verse 1 Kings 11:5 etc. Ethnic Israelites/Judahites who rejected monotheism in the B.C. period would have been culturally absorbed into the surrounding population of Canaanites or Aramaic-speaking "Syrians" (depending on the period). AnonMoos (talk) 16:01, 3 December 2008 (UTC)[reply]
Ah, but there's nothing "Jewish" about that. That's like saying that there's such a thing as Jewish sheepshearing if lots of Jews shear sheep. --Dweller (talk) 16:20, 3 December 2008 (UTC)[reply]
Before the reign of Josiah of Judah, strict monotheism had only been strongly adopted and advocated by the government/monarchy of the southern kingdom of Judah somewhat sporadically, and it was generally in the opposition in the northern kingdom of Israel (where it was advocated by some prophets in the face of royal indifference or outright hostility). At various times, there were probably a fair number of genealogical Israelites or members of the tribe of Judah who didn't think that their practices of pagan worship made them any less Israelite or Judahite than the monotheistic reformers (see Bible verse Jeremiah 44:17 for comments to this effect).
However, those Israelites and Judahites who did not sooner or later accept the leadership of monotheistic scribes and priests ultimately lost their Israelite/Jewish cultural distinctiveness, and merged into the surrounding populations... AnonMoos (talk) 02:28, 4 December 2008 (UTC)[reply]

Lawful reasons for termination of a job in united states of america

Repost from Misc

I'm not looking for legal advice, I'm just looking for a list that people have for why or why not a person can be fired. Examples are obviously race, religion, etc. Just looking for the legal document that states it or a WP list 66.216.163.92 (talk) 01:20, 3 December 2008 (UTC)[reply]

In the U.S., many people work in at-will employment situations, meaning the employer can fire them at will (unless there's a legal or contractual restriction). The article discusses some of the "why not" situations as well. --- OtherDave (talk) 01:48, 3 December 2008 (UTC)[reply]

You may want to clarify reasons someone may be terminated (fired) without the employer being required to pay compensation, (e.g., theft) often called "with cause" and other types of termination such as what might arise from a drop in business. DOR (HK) (talk) 08:35, 3 December 2008 (UTC)[reply]

Why did people vote for Ted Stevens?

I realize the elections were some weeks ago, but I was curious why so many people voted for this rascal? Any thoughts? Links to news articles with quotes from "ordinary" Alaska voters would be especially valuable in answering this question.--The Fat Man Who Never Came Back (talk) 04:24, 3 December 2008 (UTC)[reply]

Incumbents tend to stay in office because humans tend to keep doing whatever it is they are currently doing. This mindset isn't only in politics. It is throughout human behaviour. For example, the point of having super sales in U.S. stores after Thanksgiving isn't to make a lot of money on that specific day. It is to get people off their butt and in the store to do shopping. If you get them to do it one week and then somehow get them back the next week, then they will be far more likely to come back again and again and again. McDonalds has also noted this by showing that their Monopoly game increases sales right away, but the sales decline slowly when the Monopoly game is over because people got used to going there. In the end, the longer someone keeps getting voted for, the longer they will continue to get voted for. -- kainaw 04:49, 3 December 2008 (UTC)[reply]
Additionally, there was the feeling in Alaska that Stevens violated some obscure law, and saw his conviction as having little relevence to his abilities as a Senator. The theme was that he was tried in Washington, D.C., as a means to remove him from power, and the people of Alaska would be damned if some court 4000 miles away was going to tell THEM who to vote for. As far as they were concerned, Stevens did for 40 years exactly what they sent him to Washington to do, which was bring back as much cash as possible. And he certainly did that quite well, which is of course why they kept sending him back (or, almost did this last time.) As a compounding issue, Stevens is somewhat of a local hero, having been a leading campaigner for Alaskan statehood during the 1950's. He was widely considered to be very responsible for getting Alaska admitted to the union, and its really hard to unseat a hero like that; even harder than unseating a mere incumbant. --Jayron32.talk.contribs 04:58, 3 December 2008 (UTC)[reply]
to Kainaw: Well, I agree that people are often hesitant to change... though there are many exceptions, like the recent elections. You write though that In the end, the longer someone keeps getting voted for, the longer they will continue to get voted for. ... and here I disagree. That would suggest that the longer a party has reigned, the more terms of office a politician has run, the more likely they are to be reelected. All the anecdotal evidence I can think of speaks against it. At some point, people get dissatisfied with something and want a change... unless the politician has already felt the same and quit before. --Ibn Battuta (talk) 07:40, 3 December 2008 (UTC)[reply]
Other factors to consider: the generally low turnout rates in most American elections, which mean that zealous supporters, regardless of their reason, have a disproportional influence. That in turn, if you ask me, explains the saying that we're governed by the people we couldn't stand in high school. As for the benefits of incumbency, until very recently an incumbent member of Congress who chose to run was likelier to retain his seat than a member of the old Supreme Soviet. From 1976 through 2006, the re-election rate for a member of the House fell to 88% only once; from 1996 through 2006, it averaged 96%. Finally, many voters end up agreeing with the notion that "he may be a crook, but he's our crook." --- OtherDave (talk) 12:01, 3 December 2008 (UTC)[reply]
And additionally remember that if Stevens was elected and then forced to quit, that'd let a Republican governor pick the replacement (who would also be a Republican). If they voted for a Democrat, they'd end up with a Democrat, which for many was probably a worse pick than an old felon who'd be replaced in a heartbeat anyway. --98.217.8.46 (talk) 23:05, 3 December 2008 (UTC)[reply]
Actually, under Alaska law, they'd have to have a special election. (How to replace a senator who can't finish his or her term varies greatly from state to state.) After Governor Frank Murkowski appointed his daughter to a vacant U.S. Senate seat in 2002, Alaskans passed a referendum to repeal the law that allowed him to do that. What goes around, etc. --- 23:17, 3 December 2008 (UTC)
"He may be a scoundrel, but he's our scoundrel." Little Red Riding Hoodtalk 01:57, 4 December 2008 (UTC)[reply]
Thanks, as always, for the eye-opening answers. I knew that Stevens had a reputation for bringing home the pork, but did not realise what a pivotal role Stevens played in Alaska's statehood.--The Fat Man Who Never Came Back (talk) 10:23, 4 December 2008 (UTC)[reply]

Handicapped parking

Suppose you park in a handicapped spot in a private parking lot, without a handicapped permit. Could the police still write you a ticket, even though its a private lot? 97.118.238.4 (talk) 06:58, 3 December 2008 (UTC)[reply]

Quite possibly this would depend on the local laws in the state, province, or country where the parking lot is located, and also on what you mean by "private". --Anonymous, 08:00 UTC, December 3, 2008.
In the UK, these matters are handled by private security firms hired by the property owner. The penalties are usually far higher than in public areas, where the tickets are usually handed out by local council officials.--Shantavira|feed me 09:58, 3 December 2008 (UTC)[reply]
Your IP address suggests that you are in Washington State. According to this site, whose reliability I cannot vouch for, handicapped parking restrictions are enforceable by the local police in Washington State, whether or not they are in a private parking lot, but only if the space is posted with a vertical sign. That source suggests that police do not typically patrol private parking lots to ticket cars parked illegally in handicapped spaces, but it suggests that police will ticket in response to a complaint. Based on my own experience of living in different U.S. states, I think that the situation is similar in most U.S. states. Marco polo (talk) 16:19, 3 December 2008 (UTC)[reply]
To add: in many places in the US, parking in a handicapped spot without the proper tags is beyond a traffic violation. It can be a misdemeanor. --Moni3 (talk) 16:22, 3 December 2008 (UTC)[reply]
Leaving aside potential financial & legal penalties, depriving a disabled person of a designated space says volumes about the jackass doing so. Exxolon (talk) 23:24, 3 December 2008 (UTC)[reply]
Note that there is usually a (state) law requiring private parking lots to have handicapped parking stalls. The law usually specifies which lots must have them, how many stalls they need, where they must be located, etc. - the stalls don't exist just as a convenience provided by the business owner. This should go part of the way to explaining how the local police department can enforce them. Note also that some signs explicitly reference the relevant state/local law. For example, this Arizona sign [27] references "A.R.S. SEC. 28-884", presumably that section of the state law which regulates handicapped parking. -- 128.104.112.113 (talk) 02:33, 4 December 2008 (UTC)[reply]

handicpped people are always going on about how they want fair tretment and want to be viewed the same as every one else, so why do they get to park right next to the front? fuck 'em i say

They want the same access as everyone else, a level playing field, as it were. Obviously you have no experience with below-freezing temperatures, numb fingers and wheels that won't turn because you can't grip, or even how very much work it is when your whole method of locomotion depends upon your arms, or you would not begrudge an inch of distance closer to a goal. Your attitude speaks volumes about you and says nothing about those with handicaps. ៛ Bielle (talk) 00:09, 4 December 2008 (UTC)[reply]
Just to add - disability is more than just those who are wheelchair bound. The individual who made the rather disappointing comments raises the interesting debate around 'equality'. In extremely simplistic terms their proprosed solution of treating exactly the same is a type of 'equality'. That doesn't make it just/moral, but it does raise what i'd say is the conflict of logic that the pursuit for equality creates. I think it was John Rawls who said that inequality is ok provided that the inequality is used to improve the plight of the least advantaged (i'm paraphrasing as his was a discussion more around income inequality but it transfers well into most arguments of equality). 194.221.133.226 (talk) 09:23, 4 December 2008 (UTC)[reply]

@eff em, is that you George? Julia Rossi (talk) 09:48, 4 December 2008 (UTC)[reply]

And to touch on the notion of "private" parking versus "public"... the handicapped parking you see in private lots is (often?/always?) dictated by some form of local government (municipal, regional, etc.); it doesn't exist at the whim of the store or shop. Disobeying the sign is not simply going against the wishes of Wal-Mart or whoever, it's going against the wishes of the government, which is what laws usually codify. In contrast, if you stuck a wheelchair sign on your driveway and parked there, I don't think anyone would care because no law forced you to put the sign up; it's decorative. A law (or more likely, by-law) forced the shopkeeper to designate those spaces. Matt Deres (talk) 16:14, 4 December 2008 (UTC)[reply]

Bishops' cathedrals

The article on the 1947 romantic comedy film The Bishop's Wife says of the Bishop and his wife that they are "clearly members of the Episcopal Church" although the denomination is never stated. He has a wife, so rule out Roman Catholic. Certainly Episcopal works,and seems likely, but might a Methodist, Lutheran, Presbyterian or other protestant denomination in the U.S. in the 1940's have had a Bishop who sought to build a new cathedral? United Methodists today apparently sometimes call any large church a "cathedral" without any bishop in residence, similar to the "Crystal Cathedral" of Reformed Church in America's Dr. Robert Schuller (who is also apparently not a bishop). Edison (talk) 07:15, 3 December 2008 (UTC)[reply]

In the United States (and therefore in Hollywood), "the Episcopal Church" always means this Episcopal Church, even though it is true that some other churches have bishops. Marco polo (talk) 15:54, 3 December 2008 (UTC)[reply]
I think he means that the word "Episcopal" didn't actually occur in the movie, and he's wondering what other denominations in the U.S. in 1947 (if any) had married bishops and formally-designated cathedrals. (By the way, the "Crystal Cathedral" is just a grandiose name -- "does not mean it is a cathedral in the Roman Catholic, Anglican or Lutheran sense of the word" etc.) AnonMoos (talk) 16:16, 3 December 2008 (UTC)[reply]
Oops. Sorry that I didn't pay close attention. While there are other Protestant churches in the United States with bishops, the Episcopal Church is the most widely known such church. I haven't seen the movie, but there might be other clues that make it "clear" that the bishop belongs to the Episcopal Church, such as elaborate robes and other vestments, which other Protestant denominations do not use, or prayers using the Book of Common Prayer, or even very affluent congregations, since Episcopalians have long been associated with the wealthy elite in the United States. Marco polo (talk) 16:24, 3 December 2008 (UTC)[reply]

Encarta: quality? reference?

Who writes MSN Encarta? Is it considered a quality encyclopedia or not? Can you cite it in an academic paper? --Ibn Battuta (talk) 07:29, 3 December 2008 (UTC)[reply]

PS: How do you properly cite their image descriptions? (I'm not interested in the image itself, just its description.) Thanks, Ibn Battuta (talk) 07:34, 3 December 2008 (UTC)[reply]

I wouldn't cite any encyclopedia in an academic paper, assuming we're talking something higher than grammar school. Unless I was doing a paper on library science maybe.... Dismas|(talk) 08:23, 3 December 2008 (UTC)[reply]
Maybe you work in a different field. In my field, I don't mind using high-quality encyclopedias or newspaper articles to make a point, clarify a definition, provide examples, etc. if little or no scientific research exists on a topic. --Ibn Battuta (talk) 09:45, 3 December 2008 (UTC)[reply]
To answer the first part of your question, I wrote some of the articles for MS Encarta. (I probably shouldn't say which ones.) My qualifications were a doctorate in a relevant field and a history of writing on topics related to the articles. I had to submit copies of my sources. So it is reasonably reliable. Still, it is a distillation of secondary sources at best, and for an academic paper (at the university level), you should be citing primary and secondary sources directly if not doing original research. Marco polo (talk) 15:51, 3 December 2008 (UTC)[reply]

Presidential portraits

I have a couple questions about presidential portaits, so I'm just combining them...

  1. Our List of Presidents of the United States article has portraits for all the presidents except Bush and (pres-elect) Obama. So, are portraits traditionally not commisioned until after a president has left office? Just looking at the file info for Clinton's (unveiled in '04, three years after leaving office) it would suggest that is the way it's done.
  2. If the answer to that is 'yes', why?
  3. What is behind and to Clinton's left in his portrait? link to image I have an idea but I'll hide it behind that link so as not to influence anyone else's guess.
  4. And finally, is it just my crappy monitor or does it look like his jacket is made of velvet?

Thanks, Dismas|(talk) 09:07, 3 December 2008 (UTC)[reply]

Just guessing: Why would you publish a painting if you don't know what the the person looks like by the end of his office? Which makes me look forward to the first president who'll entirely change hair style and color while in office... ;o) Even more importantly, you don't really need a "loving memory" of the current president in the White House. And if a president should ever decide to post his own face to that wall, I'd suspect him to be an egomanic. And as for the delay: Even in times of digital photography, paintings tend to take a while. (And I wouldn't be too surprised if they even have to get the approval of the ex-president.) --Ibn Battuta (talk) 09:45, 3 December 2008 (UTC)[reply]
For question 3 -- those are challenge coins given to him by US servicemembers. --jpgordon∇∆∇∆ 16:50, 3 December 2008 (UTC)[reply]
Norman Rockwell's portrait of JFK was done while he was in office. --Moni3 (talk) 16:53, 3 December 2008 (UTC)[reply]
For question 4: that image has a bad case of the JPEG artifacts. That's probably the reason for the velvety look of the jacket and the appearing-disappearing pattern on the tie. I couldn't find a better image online, though, so I'm not sure. -- BenRG (talk) 02:49, 4 December 2008 (UTC)[reply]

LexisNexis

I would like to get hold of some information that I suspect is only available on the law database LexisNexis, to which I do not have access. Is there any way of getting it through Wikipedia? If there's an editor here on the desks with access to it, for example, would they be prepared to do a search for me? I'm sure I remember seeing Category:Wikipedians with access to research tools somewhere around here in the past, and I thought maybe I could approach someone there, but the category seems to have vanished (if it ever existed). Many thanks. --Richardrj talk email 09:10, 3 December 2008 (UTC)[reply]

If you provide a full citation at the Resource Request page, then someone will try to help you find the information you need.--droptone (talk) 12:34, 3 December 2008 (UTC)[reply]
Thanks, that looks like a great page. I've added my request to it. --Richardrj talk email 14:04, 3 December 2008 (UTC)[reply]

third world war 2009

how likely do u guys think it is that there will b a 3rd world war soon?

Actually, I think chances are good it'll start soon. Resources are getting more and more limited. By now, even letters are so scarce that computer users in some countries can't afford any more the "yo-"s and "-e"s, let alone capital letters and signatures. My guess is that one day not far from now, users from these disadvantaged areas will invade the more advantaged countries, which in turn will try to defend their excess letters. Speakers of wordier languages will claim that they are entitled to more letters, speakers of concise languages with short words will want to profit through trading the letters they don't use... Dark times ahead. --Ibn Battuta (talk) 09:53, 3 December 2008 (UTC)[reply]
But we did Deploy Vowels to Bosnia! -- AnonMoos (talk) 14:17, 3 December 2008 (UTC)[reply]
Tht md my dy, nnms.  :) (srry, bt ll th stndrd vwls wr tkn) -- Jckfz

The Ref. Desk is not the appropriate place for wild speculation. Mr.K. (talk) 10:47, 3 December 2008 (UTC)[reply]

haha...i don't get it...i hope...like... i can still use all the words eventhough i don't even know what half mean and i have a very limited vocabulary....lolz...that would be totally sweet... ok...but for real...do you SERIOUSLY think that we are close 2 WW III?

What do you mean by that? The usual idea of a massive nuclear war? That is really unlikely. But the "war on terror" is kind of a world war, isn't it? Adam Bishop (talk) 01:55, 4 December 2008 (UTC)[reply]

If a World War is a very large scale, extended and (nearly) unrestricted armed conflict among nearly all of the major powers, the odds of a World War in 2009 are closer to zero than to 1%. DOR (HK) (talk) 07:40, 4 December 2008 (UTC)[reply]

@ Adam Bishop: The two examples of World Wars were started by one of the major powers, which declared war and invaded other countries (especially the smaller, less powerful ones--they're of course easier to invade) for some half-baked (if at all) reasons, but with an awesome propaganda to convince its people (for a while)... oh, and that country soon got support from other major powers (until they somehow didn't agree anymore)... - Are you sure that you you want to call the "war on terror" a World War? --Ibn Battuta (talk) 04:48, 5 December 2008 (UTC) PS: At least the two previous World Wars ended when the two rulers of that country were gone...[reply]

Conscription

Did it exist in Communist Czechoslovakia? 203.188.92.71 (talk) 11:21, 3 December 2008 (UTC)[reply]

Yup. Sure was. See here. Fribbler (talk) 12:15, 3 December 2008 (UTC)[reply]

Bringing medicine into the US

If a traveller is prescribed a medicine which is not available in the US (specifically lofepramine, if an example is needed), and then enters the US with that medicine, do US travel regulations forbid them to bring it in? The Wednesday Island (talk) 16:46, 3 December 2008 (UTC)[reply]

This is a practical answer, not a legal one, and entirely WP:OR. My spouse travels with a lot of medication, some of which is tightly controlled. We also travel a lot. We have never, crossing any border anywhere, including into the U.S., had the medications get but the most cursory of glances, and the copies of prescriptions be waved off as we held them out for inspection. What the Canadian government tells you to do is keep your medications in your carry-on luggage, bring copies of all prescriptions, and keep the meds in their original containers, complete with labelling. If you originate in a country where prescriptions are not carefully controlled, however, YMMV. ៛ Bielle (talk) 17:54, 3 December 2008 (UTC)[reply]


Academic mention of Scientology's "admiration paticles"

In the Scientology and sex article, some concerns were raised about a few of the primary sources listed on the page. One such quotation is seen here:

Hubbard then went on to say that sex was an even better "communication system" for the same purposes of forced "admiration", and defined the sex act, consensual or otherwise, as "an interchange of condensed admiration particles".[1]

Does anyone know of an academic interpretation of Scientology as it relates to sex, and in particular, these "admiration particles" of which Hubbard speaks? Spidern 17:51, 3 December 2008 (UTC)[reply]

Just to let you know, there are no returns in either LexisNexis, Academic Search Premier, or Google Scholar for "admiration particles".--droptone (talk) 00:42, 4 December 2008 (UTC)[reply]

If you break it you've bought it

It is not uncommon to see sign saying "If you break it you've bought it" in shops with lots of breakable items stacked on shelves. It is my understanding that there can't be a contract for sale without an intention to buy, and the most that the shop could have is a claim for damages if they could prove negligence - certainly they could not force you to pay up on the spot. Also, they could call the police if they felt it was intentional criminal damage. Is that correct, and what "on the spot powers" do the shop have? Also, what would typically happen if the person refused to pay? Is it just that the shops do not want to claim on their insurance (or do want to pay out for insurance in the first place) - or perhaps it is just to make people more careful, while knowing that they can't enforce it? Obviously laws differ between jurisdictions, and I am most interested in the position within the UK, but would also be interested in differences abroad. Thanks WAYB (talk) 19:22, 3 December 2008 (UTC)[reply]

But there is an implicit intention to buy. Because you are not forced to enter the store, the sign says that, by entering, you agree to purchase that which you break. Wikiant (talk) 19:50, 3 December 2008 (UTC)[reply]
We really are in "legal opinion" territory here. ៛ Bielle (talk) 22:57, 3 December 2008 (UTC)[reply]
I just want to clarify that I am not asking for legal advice here, rather I am just interested in the legal dimension of this. How do I ask the question without it being classified as asking for legal advice? WAYB (talk) 22:59, 3 December 2008 (UTC)[reply]
A legal opinion is legal advice set up as a view of how something in law might be interpreted. If there were a very specific piece of law or precedent on the matter, quoting it would be a feasible enswer to your question, given the constraints of the Ref Desk. Someone may be able to point you in that direction. However, as soon as we get into how the law can be interpreted to cover a set of circumstances, you are right back to advice/opinion again. It is not the "how" of your question, but the "what". Others may disagree. ៛ Bielle (talk) 23:07, 3 December 2008 (UTC)[reply]
Oh come on, I think its perfectly acceptable to enquire as to what power shops actually have when it comes to "If you break it you've bought it". It seems like a question that can be answered here. Hammer Raccoon (talk) 02:03, 4 December 2008 (UTC)[reply]
If you can answer the question without reference as to how a law might be interpreted, then, indeed, you can do so outside of the Ref Desk constraint on offering legal advice. I can't. ៛ Bielle (talk) 03:32, 4 December 2008 (UTC)[reply]
If the shopkeeper prevents you from leaving without paying, in the U.S., he might be guilty of false imprisonment or kidnapping. If he restrains you by the use of high voltage electricity from a Taser, he might be guilty of battery. But if a policeman reaches the store before you leave, he might cite you for disorderly conduct. The shop owner should properly sue you in civil court for damages, rather than seizing money from your pocket by brute force or threats. If you accidentally or carelessly destroyed a cake in a bakery, it might be a Torte. (None of the above constitutes legal advice). Edison (talk) 05:29, 4 December 2008 (UTC)[reply]
(Consider that I am smiling as I type this.) It does might, however, constitute a legal opinion, or, rather, several legal opinions, as to what could or might happen (interpretation) based on some unspecified laws, all in a tort-uous paragraph. On the other foot, perhaps "legal opinions" can only be given by lawyers, in which case, if you are not a lawyer, then this might jus be opinions on matters pertaining to laws. (Advice -not legal- to self: quit now.) ៛ Bielle (talk) 05:44, 4 December 2008 (UTC)[reply]
(A question about the law falls into the same category as questions about anatomy - just because it is about a legal subject doesn't mean it is legal advice.) If the product is valued at over £5000, then the shopkeeper could make a citizen's (technically "any person") arrest for criminal damage (it would need to be intentional or reckless, though). If it's less than that, then the criminal damage is a "summary offence" which you can't make a citizen's arrest for, attempting to do so (or, in the above case, doing so if the damage wasn't intentional or reckless) would be false imprisonment - they would have to let you leave and just call the police. None of that depends on the presence of a sign. My "IANAL" guess would be that the sign isn't valid since there is no consideration in such a contract (I guess letting you shop there could be considered consideration, that's where you get into the realms of interpreting the law and I'm not qualified to do that). If the damage wasn't intentional or reckless, and the sign isn't valid, then the shopkeeper's only means of getting their money would be to sue in civil court for "breach of duty of care", for which they would need to prove negligence (again, that's a matter of interpretation and is beyond my (non-existent) expertise - my understanding it that it is a lesser requirement than proving recklessness, though). --Tango (talk) 11:24, 4 December 2008 (UTC)[reply]
This seems a topic much debated online[28][29][30][31][32]. There is surprisingly little consensus, other than that if you break something negligently you can be sued. Some people suggest that under civil law if a tort existed the customer would normally be liable for the wholesale price (i.e. the price the shopkeeper paid) and this sign is an attempt to extract more money by making the customer pay the retail price. --Maltelauridsbrigge (talk) 16:09, 4 December 2008 (UTC)[reply]

Can the store impose a unilateral contract? Where is the meeting of the minds? One of my favorite signs was posted in a Manhattan store. It read "unruly children will be sold as slaves." I doubt if parents who bring their children into the store are consenting that the owner may, using his discretion as to unruly, sell the children as slaves. This is a contracts question, with some elements of tort law. Some law review publication must have addressed this scenariou, even as a prank. It involves more than posting a statute without interpretation. States vary also in their interpretation of common law contracts law and statutory provisions. 75Janice (talk) 02:55, 5 December 2008 (UTC) 75Janice[reply]

The example is irrelevant however considering that slavery is illegal in Manhattan, and indeed the grater US. --S.dedalus (talk) 03:59, 5 December 2008 (UTC)[reply]

Yes, sometime during my extensive civil rights practice, including landmark cases before the U.S. Supreme Court, I figured out that slavery was now unconstitutional. The example is a hypothetical. In fact, most of these responses resemble answers on law school examinations. The only things that are missing are the cute and funny hypothetical names. 75Janice

Medieval Islam medicine

Is there a term used in Medieval Arabic for a physician/doctor? And a term used for a sort of association/society/community of such doctors? Medicine_in_medieval_Islam

--Sonjaaa (talk) 20:04, 3 December 2008 (UTC)[reply]

Doctor is "hakim" (حكيم) (which has a long i, and is related to but is not the same word as hakim with a long a, حاكم, a governor). There is also "tabib" (طبيب) but I don't know if that was used in medieval Arabic. Adam Bishop (talk) 21:20, 3 December 2008 (UTC)[reply]
Yes, it was used, more than hakim which meant much more than a mere physician. Their community was جامعه الاطبا (jaameatol-atibbaa) in a period. And hospital was دارالشفا (dar-osh-shifaa) and دارالمرضی (dar-ol-marzaa). --Omidinist (talk) 05:29, 4 December 2008 (UTC)[reply]

December 4

Barack Obama Cabinet

I am writing an article for my school newspaper and I need to find a list of Barack Obama's current cabinet. Catch is I can't use Wikipedia because the establishment thinks it is all lies. If anyone can find me a reliable website listing all of his current cabinet it would be greatly appreciated. Strifeblade (talk) 01:03, 4 December 2008 (UTC)[reply]

Thanks. That is exactly what i was looking for. Strifeblade (talk) 01:24, 4 December 2008 (UTC)[reply]

Just because you can't quote Wikipedia doesn't mean you can't use Wikipedia. If you go to Barack Obama, his cabinet nominations are listed at the bottom. Then, if you click on Hillary Rodham Clinton, you will see a section titled "United States Secretary of State nomination". Inside that are 5 links to references - any of which you may use since they are not Wikipedia. So, you can see that Wikipedia is a great way to quickly locate a lot of references, even when you can't use Wikipedia as a reference. -- kainaw 02:31, 4 December 2008 (UTC)[reply]
To counter the obvious falshood in the OP, the establishment doesn't think Wikipedia is all lies; the establishment thinks you are lazy for citing an encyclopedia in a school paper. You should always strive to cite original works where possible. Any encyclopedia, by its nature, is only a collection of paraphrases of other people's work. Use Wikipedia to give you a general overview and for understanding, but for any real writing you intend to do on your own, go find the original sources! --Jayron32.talk.contribs 05:25, 4 December 2008 (UTC)[reply]
Many people do think Wikipedia is all lies. And they're not all crazy wingnuts either. Intuitively, the idea of a wiki seems like a bad idea, (In the very early days, even Jimbo was unsure of wikis!) throw in a couple of newspaper stories about prominent errors in WP : Bingo, worse fears confirmed. APL (talk) 17:28, 5 December 2008 (UTC)[reply]
Encyclopedias and dictionaries have never been proper sources for papers. Since Wikipedia is an encyclopedia, it shouldn't be any different than other encyclopedias. -- kainaw 19:08, 5 December 2008 (UTC)[reply]

Source for Quotation from St. Basil of Caesarea

I would be very grateful if someone could identify the exact source for the following prayer from St. Basil the Great:

O God, enlarge within us the sense of fellowship with all living things, our brothers the animals to whom Thou gavest the earth as their home in common with us. May we realize that they live not for us alone, but for themselves and for Thee and that they love the sweetness of life even as we, and serve Thee better in their place than we in ours. For those, O Lord, the humble beasts, that bear with us the burden and heat of the day, and offer their guileless lives for the well-being of mankind; and for the wild creatures, whom Thou hast made wise, strong, and beautiful, we supplicate for them Thy great tenderness of heart, for Thou hast promised to save both man and beast and great is Thy loving kindness, O Master, Savior of the world.

There are many quotations of this prayer on the web, but none seems to give its source. One person ascribed it to the Liturgy of St. Basil, but I could not find it therein. I would like to use the quotation and cite the source, and would ideally therefore very much appreciate a clear identification of the original work, with chapter (and lines, where appropriate), as well as the reference in Migne, if known.

Many thanks for your kindness. Scrutor (talk) 02:16, 4 December 2008 (UTC)[reply]

Are you sure that's Basil? Sounds more like Francis of Assisi (although his blessing for animals seems to be somewhat different). Adam Bishop (talk) 03:08, 4 December 2008 (UTC)[reply]
Hmm, this webpage has a reference to "Journey to Heaven: Counsels on the Particular Duties of Every Christian, by St. Tikon of Zadonsk, transated by Fr. George D. Lardas, Holy Trinity Monaster, Jordanville, '91". Maybe that will have a reference, if you can find it (there's no preview on Google Books unfortunately). Adam Bishop (talk) 03:52, 4 December 2008 (UTC)[reply]
Similar to: Rauschenbusch, Walter. (1910) "For this World." Prayers of the Social Awakening. pp. 47-8.—eric 09:20, 4 December 2008 (UTC)[reply]

Although the prayer does sound "Franciscan," there are MANY references to it on the web as coming from St. Basil. I am hoping that someone familiar with his works will be able to identify its source. —Preceding unsigned comment added by Scrutor (talkcontribs) 21:31, 5 December 2008 (UTC)[reply]

Which U.S. Presidents have been the subject of a major movie/film?

Which U.S. Presidents have been the subject of a major movie/film? And, please list the titles of those movies.

i.e. JFK, Nixon, [recently released]

Dubya George W. Bush You might get a better answer to this on the Entertainment Desk ៛ Bielle (talk) 03:24, 4 December 2008 (UTC)[reply]
That's W., at least on this continent. I would add Sunrise at Campobello (Franklin Roosevelt) and Give 'em Hell, Harry! (Truman). --Anonymous, 08:01 UTC, December 4, 2008.
There was a miniseries, "Backstairs at the White House," about the servants at the White House which featured the tenures of many 20th century presidents and portrayed them: Taft, Wilson, Harding, Coolidge, Hoover, Franklin Roosevelt, Truman, and Eisenhower. Another similarly themed movie about a servant at the White House will cover a later 34 year span, 1952 (Eisenhower) through Kennedy, Johnson, Nixon, Ford, Carter, and Reagan. [34] says every U.S. president has been featured in some film, with Fillmore (The Monroe Doctrine (1939)) and Buchanan ("The American President" (2000)) bringing up the rear at one movie each. Lincoln has been the subject of 210 movies or appearances. Nixon ("All the President's Men," (1976), "Nixon" (1995), Washington (119 appearances), Jefferson (81 appearances), Grant (73 appearances), and Franklin Roosevelt (69 appearances), as well as George "W" Bush (67 appearances). Edison (talk) 05:15, 4 December 2008 (UTC)[reply]
The question asked about presidents that were the subject of movies, not about presidents that appeared in movies, let alone TV productions. --Anonymous, 08:01 UTC, December 4, 2008.
Wag the Dog and George H.W. Bush, although it was not made explicit. Surely there was a war movie about Eisenhower.--droptone (talk) 12:41, 4 December 2008 (UTC)[reply]
Yes, of course. Henry Grace played him as General Eisenhower in The Longest Day. That movie still "wounds my heart with a monotonous languor". -- JackofOz (talk) 12:58, 4 December 2008 (UTC)[reply]
The Longest Day is probably not the type of film the questioner is asking about, since Ike is not really the subject of the film, and of course he wasn't the president yet. If the question is indeed meant to include films about presidents before they were president, we can add to the list things like Young Mr. Lincoln, and, if TV movies count as "major", movies such as The Crossing and Ike: Countdown to D-Day and Warm Springs. —Kevin Myers 14:06, 4 December 2008 (UTC)[reply]
We don't have a list of movies, but we do have a list of actors who played President of the United States. Not all of these are centered on the president in question and many are fictional portrayals, but it a place to start. --—— Gadget850 (Ed) talk - 14:33, 4 December 2008 (UTC)[reply]

See Truman (film), Wilson (film) and Old Hickory -- Mwalcoff (talk) 00:07, 5 December 2008 (UTC)[reply]

Existentialism

Can anyone describe the fundamentals of existentialism in under fifty words? The article on it is absolutely confusing. --hello, i'm a member | talk to me! 03:33, 4 December 2008 (UTC)[reply]

Existentialism is a broad set of philosophical schools which hold that certain basic ideals. Its a really huge part of philosophy, with MANY diverse schools of philosophy being called "existential" but they all share certain basic commonalities.
  • While most other schools of thought deal in theoretical constructs used to explain human existance and relationships (think of Hobbes' Leviathan theory or Rousseau's social contracts), existentialism is always firmly grounded in existence, that is it confines itself to actual human interactions and actual human conditions rather than constructed or theoretical ones. This is contained in the idea that "Existence precedes essence", which is sort of the philosophical equivalent of the "nature vs. nurture" debate. This also means that things exist outside of our definitions of them, however nothing has any inherent meaning. A table exists, but it does nothing except exist. It is only a table, rather than a random collection of wood, because in my mind I assign it meaning.
  • Most existential philosphies deal with the problems of internal conflict or tension or angst of some sort. The basic problem for existentialism is that free will by its nature creates stress, and how people deal with that stress. There is also the tension between facts of your life you cannot control (called facticity) and your responses to these facts, which of course you are always free to choose. Your conflicting desires to both conform to the facts of your existance, and to break free from them, is a major source of angst. Since nothing in life has any meaning beyond what we assign to it, it is the neverending search to apply meaning to our ultimately meaningless lives which also leads to angst.
  • Existentialism is also usually about the search for the authentic life, whatever that may mean. We should live according to our nature, and not according to externally imposed expectations on our behavior.
  • Existentialism holds that people make decisions not based on rational thought, but instead all of our decisions are based on attempts to reduce the angst in our lives, as noted above. Whether we choose to succumb to the pre-existing facts of our condition, or whether we choose to break free from them, has little to do with whether our actions will actually make our lives better in a measureable way, but rather we make our choices by taking the course of action that will reduce our internal stress. Thus, people will tend to stay in rediculous situations out of comfort, rather than to leave out of reason. Why would a smart youth in a Western nation with access to quality public education choose to remain in a ghetto and sell drugs, and probably get shot and die, when there are easily availible rational means to get out of that situation? Because leaving the situation would raise his angst level, so he never sees it as a viable option.
That's way more than 50 words, but its a complex series of thoughts, so I tried to hit the highlights.--Jayron32.talk.contribs 05:19, 4 December 2008 (UTC)[reply]
  • From a "what is your world view" quiz (I was judged to be an existentialist): "Existentialism emphasizes human capability. There is no greater power interfering with life and thus it is up to us to make things happen. Sometimes considered a negative and depressing world view, your optimism towards human accomplishment is immense. Mankind is condemned to be free and must accept the responsibility."Mieciu K (talk) 05:22, 4 December 2008 (UTC)[reply]
What makes you so special? It's all a big nothing. You die in your own arms.Livia Soprano to her grandson, exhibiting the bitter side of existential thinking. Julia Rossi (talk) 08:09, 4 December 2008 (UTC)[reply]
Sartre claims that the one thing that existentialists all have in common is that subjectivity is the starting point, or that existence precedes essence. Whatever that means. 128.192.81.83 (talk) 18:20, 4 December 2008 (UTC)[reply]
Sometimes it helps to look at its opposite philosophy to see what it is not, viz essentialism is a belief that things have a set of characteristics that make them what they are, and it's the job of culture to instill "the cultural norm", in other words, essentialism is about essence preceding existence. Both these propositions are a train it's hard to get off in a hurry, Julia Rossi (talk) 09:01, 5 December 2008 (UTC)[reply]

Short selling and shareholder rights

So when you sell short someone else's stock shares that you borrowed, and if there are shareholder rights that come with the shares in this stock (I dunno, e.g. you can vote on company decisions or something), do you then deprive the original shareholder of these rights? Otherwise, both the person you borrowed shares from and the person you sold it to will expect the rights from those same shares, which can't be possible, right? --128.97.245.18 (talk) 08:39, 4 December 2008 (UTC)[reply]

According to Investopedia, the person the short seller sold the shares to becomes the owner of those rights so the original shareholder (i.e. the lender) is deprived of them. See the link for details. Zain Ebrahim (talk) 09:03, 4 December 2008 (UTC)[reply]
Stock lending agreements usually include provisions to compensate the lender for any benefits that they would otherwise have lost during the period of the loan. For example, the borrower will agree to pay the lender an amount equal to any dividends that the are paid on the stock during the period of the loan - this is called a "manufactured dividend". Voting rights are more difficult to compensate for, as only one party can vote the shares. However, if the lender knows there is an AGM coming up and especially wants to exercise their vote, they will recall the loan before the AGM. If they don't want to exercise their vote, they may be able to negotiate a higher interest rate with the borrower, as the supply of stock available for lending may reduce as the AGM approaches. Gandalf61 (talk) 09:41, 4 December 2008 (UTC)[reply]

neil rosenstein doctor surgeon/genealogist of elizabeth new jersey

what is the current up date on this gentleman's status. is he no longer a doctor and if not why? —Preceding unsigned comment added by 196.25.255.194 (talk) 12:31, 4 December 2008 (UTC)[reply]

Maybe he's too busy researching his family history to do any doctoring.  :) -- JackofOz (talk) 12:59, 4 December 2008 (UTC)[reply]
It appears both his New York medical license [35] and his New Jersey medical license were voluntarily surrendered [36] in 2008. The reasons are cited in the linked pages. - Nunh-huh 21:54, 4 December 2008 (UTC)[reply]

See the whole world with his naked eye

Who is the first person to see the whole world (planet earth) with his naked eye? 122.107.203.230 (talk) 12:36, 4 December 2008 (UTC)[reply]

Yuri Gagarin was the first human to reach space and the first human to orbit the Earth, so I would presume he was the first.--droptone (talk) 12:45, 4 December 2008 (UTC)[reply]
Nobody's seen the whole thing at once; however, the Apollo 8 astronauts were the first to see the maximum possible amount at one go (Gagarin's orbit wasn't high enough to see a full hemisphere). — Lomn 15:48, 4 December 2008 (UTC)[reply]
Wouldn't you need to be at infinite distance to see a whole hemisphere? The Apollo 8 astronauts presumably saw something very close to the whole hemisphere, but there would have been a ring around the edge they couldn't see. --Tango (talk) 18:53, 4 December 2008 (UTC)[reply]
Well, yes. Point is, if "saw as much of the world as possible at a glance" is the criteria, then any of the lunar Apollo missions would have seen substantially more than other astro/cosmonauts. Similarly, if the criteria is "saw the whole world, though over a period of time", Gagarin's orbit might not have been sufficient to see everything. The lunar missions would have stood a significantly better chance. I don't know how to go about verifying that premise, however. — Lomn 19:24, 4 December 2008 (UTC)[reply]
Throwing together some very back-of-the-envelope calculations, Gagarin might just barely have been able to see the whole surface of Earth, but conditions would have needed to be ideal. At an altitude of 200 miles (Gagarin's apogee), an orbital inclination of about 69° is needed to see the poles. Gagarin's inclination was 65° -- short, but given my rough calculations, that's within the margin of error. However, his orbit was highly elliptical -- perigee was 112 miles. So he'd have had to reach apogee near the extremes of inclination, probably on multiple cycles, to see the full sum of polar regions. It's unlikely.
Apollo lunar astronauts, on the other hand, might have one pole substantially obscured due to the Earth's axial tilt, and I'll get a headache trying to determine the math for that one.
Another major candidate set, that of astronauts on space stations, appears to fail due to insufficient orbital inclination. Skylab, Mir, and the ISS all have ~50° inclinations, far short of what their ~200 mile altitudes would require. — Lomn 19:46, 4 December 2008 (UTC)[reply]
Let's not forget atmospheric refraction! —Tamfang (talk) 20:27, 4 December 2008 (UTC)[reply]
Even by the interpretation where you see the entire surface of Earth in sections, Gagarin is definitely wrong. He only flew once (the Soviets didn't want to risk losing their hero in an accident) and only made one orbit. So the parts of the Earth that were not near his orbital plane at that time of day would not have been visible. The first person to fly multiple orbits was Gherman Titov in the following Soviet mission; he stayed in orbit for a full day. But if these missions were only high enough above ground to see one of the poles when at apogee, then other pole would not have been visible.
I think the Apollo 8 astronauts are the best answer in what I think is the spirit of the question, but I have no idea which of the three men was the first to actually look out of the window and see that view. --Anonymous, 05:17 UTC, December 5, 2008.

Richard Nixon

Is this true that when Republican Richard Nixon was the president of U.S., he agreed with Pakistan about not recognizing Bangladesh in the 1970s? —Preceding unsigned comment added by 142.204.74.211 (talk) 15:37, 4 December 2008 (UTC)[reply]

Yes, that's correct. See Bangladesh_Liberation_War#USA_and_USSR. --Sean 19:31, 4 December 2008 (UTC)[reply]

What's wrong with this ballot?

In the recent Minnisotta senate election, this ballot was being disputed by Al Franken. The challenge reason (if I'm reading it correctly) is "Not Coleman, is defaced." Regardless of the validity of the challege, what exactly was Franken claiming is wrong with this ballot? The voter obviously voted for Coleman nor do I see where the ballot was defaced.

http://senaterecount.startribune.com/media/ballotPDFs/yellowmedicine_CanbyW1_challengedballot2.pdf

216.239.234.196 (talk) 18:24, 4 December 2008 (UTC)[reply]

I guess they mean the way the ovals were scribbled over rather than filled in neatly. I don't know how the US works, but in the UK a vote is valid as long as it is clear what the intention of the voter was, which it clearly is in this case. --Tango (talk) 18:52, 4 December 2008 (UTC)[reply]
I don't know Minnesotan electoral law, but the fact that some of the scribble has made its way into the Al Franken oval would invalidate this ballot in some places. Algebraist 19:08, 4 December 2008 (UTC)[reply]
I wonder if they might be contending that the voter may have started filling in the ovals, then changed their mind and scribbled out their choices? As they haven't marked any other choices and clearly placed the paper in the ballot box, it seems a very weak suggestion, but as I understand it, the general idea is that the candidates challenge any ballot where there could be any possible case to invalidate it. Warofdreams talk 20:37, 4 December 2008 (UTC)[reply]
Some of the ovals on the back are more properly filled in. So I suppose you could argue that the guy voted for coleman, and then changed his mind and scribbled it out. So it is "not" a vote for "Coleman". Bit of a stretch though. APL (talk) 20:10, 5 December 2008 (UTC)[reply]
I agree with the previous responders. It's appears that Franken's team is claiming that the voter scribbled out his choices, which makes the ballot "defaced" and effectively invalid. What is perfectly clear to me is that the person who filled out this ballot is a sloppy idiot who can't follow simple instructions.  :-) --Zerozal (talk) 21:20, 4 December 2008 (UTC)[reply]
Or just old or disabled or has a tic or whatever. But probably technically invalid, which is as good a reason as any to object to it if you are trying desperately (as both sides are) to invalidate as many ballots for your opponent as possible. No doubt the Coleman camp is doing exactly the same thing, as they are producing even more "contested" ballots than Franken's camp is, at the moment. The margin of votes to determine one or the other is at the moment less than 100 which is close enough to try and pull out all the stops. (Personally I think it's close enough for a re-vote, as it is so close that there is no margin of error for it, but whatever. ) --140.247.11.38 (talk) 23:13, 4 December 2008 (UTC)[reply]
The vote for alderman looks weird too. Why vote for "Write-In" and then not write anyone in? APL (talk) 20:06, 5 December 2008 (UTC)[reply]
Well it does say, 'if any'. If I were voting there and didn't read the instructions, and wasn't otherwise aware of practice, I might presume that if you wanted to not vote you choose write in, if any, but don't write in. What makes it a odd is he/she did not vote at all for others. Perhaps he/she didn't like the candidate but wasn't aware of any other write-in candidates and either wanted to express his/her displeasure or thought it would make a difference Nil Einne (talk) 18:00, 6 December 2008 (UTC)[reply]

Irish Law

Describe the important events which lead to the adoption of the 1922 Irish Constitution. —Preceding unsigned comment added by 89.100.234.21 (talk) 18:43, 4 December 2008 (UTC)[reply]

We don't answer homework here; check the index of your textbook. Matt Deres (talk) 18:48, 4 December 2008 (UTC)[reply]
Also check out Constitution of the Irish Free State. Algebraist 19:06, 4 December 2008 (UTC)[reply]
We also have an article on the History of the Republic of Ireland, but it should be the best two references for answers to your question are going to be: 1) The textbook your teacher gave you at the start of the class and 2) the class notes you wrote down the day your teacher taught you this. --Jayron32.talk.contribs 19:57, 4 December 2008 (UTC)[reply]
Make sure you spell the question correctly when you write your answer: "led to the adoption". Itsmejudith (talk) 21:00, 4 December 2008 (UTC)[reply]
For a start, the Easter Rising and its aftermath, the Conscription Crisis of 1918, the Irish general election, 1918, the Irish War of Independence, and the Partition of Ireland. But (also for a start) see Norman invasion of Ireland, Tudor reconquest of Ireland, Cromwellian conquest of Ireland, Williamite War in Ireland, Irish Famine (1740–1741), Irish Rebellion of 1798, Great Famine (Ireland), Irish Famine (1879), and Local Government (Ireland) Act 1898. Strawless (talk) 23:11, 4 December 2008 (UTC)[reply]

Paper on Cotton in the "India Trade"

I am currently gathering Arabic, Jewish, Indian, and Chinese sources for a paper I am writing. It will briefly detail the history of cotton trade in the middle east and Asia and focus on how the lack of large scale cotton production in Song Dynasty China might have led Muslim and Jewish "India Traders" to bring the product to China. My initial research shows cotton was probably grown in China during the ancient dynasties, but remained rare. Foreign cotton was introduced into China during the Tang and was not produced on a large scale until the late Song Dynasty.

I already have a good bit of Jewish sources via books on Cairo Geniza letters by S.D. Goitein, but I am falling short on all of the others. The Cotton article holds little if any material that I can use. Hopefully there are some people here that know of some good sources on the subject. I would also like to give examples of certain foreign merchants that came to China to find their fortune (not necessarily for the pillar of travel like Ibn Battuta). I think several good examples would help support my theory. Thanks in advance for any help provided. --Ghostexorcist (talk) 19:33, 4 December 2008 (UTC)[reply]


Seek knowledge even as far as China

Muslims have a Hadith that roughly translates to "Seek knowledge even as far as China". Does Judaism have a similar saying? A fellow member of WP:WikiProject Judaism told me the "Book of Proverbs is dedicated almost exclusively to the importance of acquiring knowledge and wisdom. To my knowledge, the Ancient Hebrews probably did not know of China." But according to Donal Leslie's Survival of the Chinese Jews, "A few Greek writers used the term Thinae, as opposed to the more common Seres" to designate China. I imagine the Jews knew of it if the Greeks did. I understand Greek was one of the languages spoken by the ancient Jews. --Ghostexorcist (talk) 20:59, 4 December 2008 (UTC)[reply]

I would imagine that most western peoples knew of China. Rome certainly did, the Greeks established nations at China's doorstep (i.e. Bactria). Chinese goods could be found in most parts of the Old World, and western goods found their way to China throughout most of recorded history. The Hebrews in ancient times were not very populous people (despite the millions claimed in the Bible, most historical estimates make the estimate population of the people that would become the Jews MUCH smaller than that). For that reason, I would not be surprised if there is no evidence of direct contact between the ancient Jews and Chinese in the way that Greeks and Chinese or Ethopians and Chinese likely had contact; however contact does not mean knowledge. As Jews had frequent contacts with people who themselves likely had contact with the Chinese, its unlikely that the China as a nation was entirely unknown to the Jews. --Jayron32.talk.contribs 21:20, 4 December 2008 (UTC)[reply]
Sometimes ancient Greeks and Romans refered to the "Seres" as the source of silk traded through a northern route, and the "Sines" as something approached through a southern sea route beyond India, without being certain whether the two were the same or different. But I'm not sure what this has to do with the possible existence of an analogous Jewish saying, which wouldn't necessarily have to refer to China at all... AnonMoos (talk) 21:45, 4 December 2008 (UTC)[reply]
Ancient Jews did speak Greek — virtually everyone around the eastern Mediterranean did, to an extent, for several centuries — but this was after Alexander's conquests. What do you mean by "ancient", however? If you mean in the time when Hebrew, rather than Aramaic, was the typical cradlespeech, the period you mean extends (if I remember right) only a little while after the Exile, long before the Greeks conquered the region. An occasional Jew might have travelled far enough to the east to have learned of China, but I seriously doubt that it would have been known by an established Jewish community. If you're meaning the pre-exilic period when most or all of Proverbs was composed (after all, Hezekiah was a century before the Babylonian conquest), I would say it virtually impossible: given that 2 Kings 20.14 speaks of Babylonian envoys to Hezekiah as being "from a far country, from Babylon" (NJPS), I can hardly imagine even well-educated residents of the state of Judah having a chance of knowing about China. Nyttend (talk) 02:27, 5 December 2008 (UTC)[reply]
Well, the term "far country" could just mean "not next door". It doesn't mean neccesarily "the farthest place we know about." Even pre-exilic Jews had contact with black Africa, such places as Ethiopia. Its about 2000 miles from Isreal to the horn of Africa, while its about 3000 miles from Isreal to Tibet. I wouldn't put it out of the realm of possibility that the Jews had knowledge of China, even if they didn't have contact with the chinese. --Jayron32.talk.contribs 05:06, 5 December 2008 (UTC)[reply]
Back to the OP, it should be noted that Islam was founded in the 600's AD; Jews in 600 AD can with certainty be said to have known about china.--Jayron32.talk.contribs 05:06, 5 December 2008 (UTC)[reply]
For general western contact with China, we have Sino-Roman relations. Some article also mentions that Buddhists may have been present in ancient Athens, but I can't find it at the moment. Adam Bishop (talk) 15:03, 5 December 2008 (UTC)[reply]
I doubt that there were practicing Buddhists in Athens, but there WERE Greeks in Bactria and India who converted to Buddhism. See Greco-Bactrian Kingdom, and Indo-Greek Kingdom and expecially Menander I.
Ok, back to the topic at hand. Does Judaism have a saying similar to the Muslim hadith mentioned above? --Ghostexorcist (talk) 19:50, 5 December 2008 (UTC)[reply]

Finding a blog post on immigration law

This is a far-out request, but I figure if anyone can help find it, it's Wikipedia. Two years ago, while researching some cases on British law, I stumbled across a blog written by an anonymous lawyer in the US. One of the blog's most visited and well-known posts was the lawyer's story of defending his foreign-born (I believe European) wife from deportation. It dealt with the corruption and opacity of the American naturalization/immigration process, and related how he had to fight tooth and nail, ultimately arguing single-handedly in a federal court against lawyers from I think DHS or INS. The judge ruled in his wife's favor, and forced the government to let her stay. That's the rough outline of the story. I've done my darndest to find the story using Google, but I have come up with nothing so far. Does anyone else remember this story, or have an idea of how I might find it? Johnleemk | Talk 23:11, 4 December 2008 (UTC)[reply]


December 5

US government: can the President fire the Vice-President?

Let's say that the President of the United States was unhappy with the Vice-President's performance, or thought the VP had acted unethically. Does the President have the authority to fire and replace the VP, and if so, whose approval and what legal processes are necessary to carry this out? 69.224.113.5 (talk) 01:01, 5 December 2008 (UTC)[reply]

Not sure, but according to the Spiro Agnew article, Agnew later said that Nixon threatened to assassinate him, which is probably on the extreme end of the scale of authority. --98.217.8.46 (talk) 02:00, 5 December 2008 (UTC)[reply]
There's nothing in the Constitution about a way to get rid of a Vice President in term, other than impeachment by the House of Representatives and conviction by the Senate. This is in significant contrast to members of the Cabinet, who fall under the Constitution's provision that the President may appoint high-ranking executive officers with the Senate's confirmation. Because the Vice President is elected for a four-year term — not appointed — this provision doesn't apply. You say "had acted unethically": although it would be politically devastating, I suppose the President could urge the Congress to impeach and to convict the Vice President. Other than that, the only way to get rid of the Vice President legally is to wait until it's time to run for reëlection, going with someone other than the sitting vice president on the ticket. For an example of this, see Henry A. Wallace, one of three men to be Vice President during Franklin Roosevelt's time in office. Nyttend (talk) 02:11, 5 December 2008 (UTC)[reply]
Remember that in the original constitution, the VP was the person the president beat in the election. Thus giving the president the power to sack him would be rather silly. Algebraist 02:13, 5 December 2008 (UTC)[reply]
Good point :-) By the way, the President does have the power to replace the Vice President: according to the 25th Amendment, if something happens to the Vice President, the President may nominate someone to the post, who will become Vice President upon confirmation of both houses of Congress. Nyttend (talk) 02:29, 5 December 2008 (UTC)[reply]
Lots of presidents did not get along with their veeps, Kennedy and Johnson never really clicked, and Reagan and Bush Sr. came from very different political mindsets, and were not known to get along very well. Bush Sr. was definitely against Reagan's economic policies; he famously called them "voodoo economics". Then again, it wasn't long ago that the veeps ONLY important job was waiting for the president to die. John Nance Garner once said that the job of vice president wasn't "worth a bucket of warm piss". Constitutionally, the veep has only one job; to break tie votes in the Senate. Given that the veep really doesn't have any real purpose beyond what the president delegates to him (usually he's merely a mouthpiece for presidential policies and little more) its really not that big of a deal should the president not like him. The current situation, where the Vice President acts as the Éminence grise in the Bush Administration is an entirely recent development. --Jayron32.talk.contribs 04:53, 5 December 2008 (UTC)[reply]
A President definitely cannot "fire" a Vice President. The VP would have to resign or be impeached. But the President directs the Executive Branch. He could give the VP no access to Cabinet meetings or secret documents or policy discussions. I'm not sure if specific legislation gives the VP certain funding for operation of his office. If not, the VP could be denied staff, shut out of the West Wing and otherwise humiliated. The VP would still apparently have the right to preside over the U.S. Senate. His official residence and salary would not be subject to arbitrary changes by a president. See John C. Calhoun for an example of a VP at odds with a president. Edison (talk) 20:07, 5 December 2008 (UTC)[reply]

Old musical time signatures

I found a scan of an old 1680 encyclopedia on the Internet, and was randomly looking through it when I noticed something very interesting.

Look at where it discusses time signatures: there are three marks with some sort of common-time symbol in them. The text identifies them as "Characters that distinguish the Movements in Common Time, the first [common time] implying slow, the second [cut time] brisk, the third [a sort of reversed cut time] very quick.

My question is: a) When did using time signature as an indication of tempo instead of rhythmic accent fall out of favor, and b) is there any modern descendant to the reversed cut time symbol, like common time now means 4/4 and cut time means 2/2? 69.177.191.60 (talk) 01:34, 5 December 2008 (UTC)[reply]

It was only very gradually that indications of tempo became separate from what we now call the time signature. I think the earliest occurrence of words to indicate tempo was in Luis de Milán in 1536, but became increasingly common during the 17th century. Still, the symbols for time signature never completely lost their association with tempo (they still have it -- "cut time" certainly implies fast.) I don't know of any contemporary survival of the reversed cut-time symbol (those symbols themselves are survivals of the mensural symbols of the Middle Ages -- e.g. the tempus perfectum, a circle, triple meter; and tempus imperfectum, a half-circle, two-to-a-bar). Antandrus (talk) 03:24, 5 December 2008 (UTC)[reply]
According to our article on time signatures, indicates some older symbols that were used to indicate tempo. In the last section on proportions, it notes that the "forward c crossed " indicated a tempo modulation to double speed, while a "o crossed" represented a tempo modulation to triple speed. Perhaps the "reversed c crossed" is a variant of the "o crossed" noted in our article... --Jayron32.talk.contribs 04:43, 5 December 2008 (UTC)[reply]

By the way, that is the Cyclopaedia, or Universal Dictionary of Arts and Sciences of Ephraim Chambers, published in 1728, not 1680. Strawless (talk) 11:23, 5 December 2008 (UTC)[reply]

Looking for Chinese Political Economy courses in English

Does anyone know where I can find online textbooks or lecture notes from a Chinese university on modern Chinese economic thought?Something like this: http://64.233.169.132/search?q=cache:O6lBkOpjb2MJ:www2.jci.jx.cn/2006/zzjjx/images/4.1.2%2520The%2520outline%2520about%E3%80%8APolitical%2520Economics%E3%80%8B.doc+%22state+monopoly+capitalism%22+%22socialist+market+economy%22&hl=en&ct=clnk&cd=1&gl=us&client=firefox-a

What are the mainstream economic views currently held in China, are there any resources on that topic? How have they integrated Marxism with Neoclassicalism? --Gary123 (talk) 05:40, 5 December 2008 (UTC)[reply]

If you are looking for very academic analysis, my advice is to look to scholarly journals rather than textbooks. The reason is that the Chinese leadership isn’t all that concerned with neoClassical vs. Marxism any more. Deng Xiaoping’s cat killed off the Marxist rat once and for all (we hope). Try China Quarterly, The China Journal, China Economic Journal, China Economic Review, China and World Economy or Journal of Chinese Economic and Foreign Trade Studies. DOR (HK) (talk) 10:03, 5 December 2008 (UTC)[reply]

I know the legal status was different for a woman in the 19th century and before, depending if she was married, unmarried or a widow; a married woman was under the legal guardianship of her husband. As for an unmarried woman, she was under the guardianship of her nearest male relative all her life (although this may differ between the european countries), and a widow was of legal maturity just like a man. Wat was the legal status for a divorced woman in, say, the 18th century? Was she of legal age, or under guardianship? I know, of course, that divorces was unusual in those days, but I am talking about the times when they did happen. I hope someone can answer me! Thanks!--85.226.45.121 (talk) 11:40, 5 December 2008 (UTC)[reply]

In the common law of England from ca. the 17th-century on, an unmarried woman was not "under the guardianship of her nearest male relative all her life". After she turned 21, an unmarried woman could make contracts, keep her earnings, and marry whom she chose. (See Coverture.) However, in wealthy families, there were frequently a variety of trust arrangements, which removed from women direct control over money or property they had inherited (though it was still supposed to be used for the benefit of themselves and their descendents)... AnonMoos (talk) 16:31, 5 December 2008 (UTC)[reply]
Thank you, I was unsure. But my question was about divorced women. Do you know that answer? By the way, it was interesting to know that unmarried women were seen as adults in England; in most countries in Europe, they were not, so England must be an exception to that rule. In my own country, (Sweden) they were minors until 1858. Can you tell me about the divorced women? I would be grateful. My question refers to the countries in Europe, were divorce were legal before ca 1850. --85.226.45.121 (talk) 21:47, 5 December 2008 (UTC)[reply]
The answer to the specific question you asked would be long and boring, and it's difficult to answer it with great confidence unless you're a professional legal historian. Before the Matrimonial Causes Act 1857, a divorce which allowed someone to lawfully remarry while the ex-spouse was still living (and for the children of such a marriage to be considered legitimate) could only be obtained in England through a long convoluted legal and parliamentary process which only a few rich people could afford. Such a full divorce presumably would have re-established a woman's feme sole status (unless parliament said otherwise), but was quite rare. For couples who didn't want to go through the whole long and debilitating rigamarole (or for whom it wasn't realistically feasible), there was a form of legal separation which was sometimes called "divorce" (but which didn't allow lawful remairriage). To what degree a separation gave a woman effective feme sole rights was probably a quite complex question; certainly Caroline Norton found out at one point that a separation seemed to diminish her estranged husband's duty to support her economically without necessarily correspondingly diminishing his rights under coverture to appropriate any earnings she made... AnonMoos (talk) 05:57, 6 December 2008 (UTC)[reply]

Every little helps

What advertising company does Tesco currently use? Donek (talk) 12:22, 5 December 2008 (UTC)[reply]

I can't answer your question, but am I right in assuming you mean Tesco in the UK, and not say Malaysia or China? While it's possible they use the same company universally, I doubt it personally Nil Einne (talk) 14:42, 6 December 2008 (UTC)[reply]
Sir Frank Lowe's company Red Brick Road are handling their UK advertising, including their new Xmas ad[37], though they will contract some work out. RBR's website has more info on their campaign. Other press stories:[38][39][40] --Maltelauridsbrigge (talk) 16:07, 6 December 2008 (UTC)[reply]

Rational action

If an action is rational in a given circumstance if it is the action most likely to achieve an agent's ends, given the information available to him, can two differing actions be rational in identical circumstances? Has anyone written anything on this? Thanks in advance 62.30.249.131 (talk) 16:33, 5 December 2008 (UTC)[reply]

Buridan's ass is relevant. Algebraist 16:46, 5 December 2008 (UTC)[reply]
I wouldn't say a rational action is one most likely to achieve your goals, it's one that maximises utility (value, usefulness). Say you see your friend on the other side of the road and want to get over there before they leave and there is a pedestrian crossing nearby. You have two options, go to the crossing or cross where you are. The latter will be quicker so makes it more likely that you will get there before your friend leaves, but the former is safer. The rational choice may well be to take the slower route and risk missing your friend in order to avoid a large, although unlikely, loss (your life). Given that definition of a rational action, you can very easily get differing conclusions on what is rational with them all being just as logically valid - you just have to start from a different value system. For example, is is rational to have an abortion? The answer depends on your values (do you value the life of the fetus more or less than your convenience/health/whatever other reason you have for not wanting the child?). --Tango (talk) 16:51, 5 December 2008 (UTC)[reply]
It is often rational to throw a dice to decide what to do, see Game theory. Also different people in the same circumstances can have different priorities. Personally I'm rather surprised by the way people seem to have quite different motivations and yet come to the same conclusions so often, but that's practically the opposite of what you're asking. Dmcq (talk) 20:29, 5 December 2008 (UTC)[reply]

Public Transportation between Springfield Mass and Hartford Conn.

I live in the Springfield area and I wish to take a job in Hartford. Unfortunatly, I do not drive a car yet I still wish to commute from Sprinfield to Hartford. Is their any low cost public transit that connects Springfield and Hartford? Are their any private companys that conduct relativly cheap van service between hartford and Springfield? I would really appreciate any information I can get on this subject matter. —Preceding unsigned comment added by 68.118.255.30 (talk) 18:39, 5 December 2008 (UTC)[reply]

There are services that likely provide transportation to Bradley International Airport from both cities; but given that they are in different states, I am not sure that there is a single agency that provides public transport between the two cities directly... --Jayron32.talk.contribs 18:58, 5 December 2008 (UTC)[reply]
Apparently, you CAN use two public services to get between Springfield and Hartford. Connecticut Transit has a connection in Enfield, Connecticut to Pioneer Valley Transit Authority. No guarantees on how convenient the connection or travel times will be to your situation. The articles above have links to the websites of these public bus services. Cheers! --Jayron32.talk.contribs 19:06, 5 December 2008 (UTC)[reply]
(after EC) The number 5 Hartford Express route map has a note at the northern terminus stating "Connections available with PVTA Route #5 to Springfield at Mass Mutual. Call 413-781-PVTA or visit www.pvta.com for information.". Google turned up a ton of other rideshare and carpool websites. --LarryMac | Talk 19:10, 5 December 2008 (UTC)[reply]
This map shows that the CT Transit 5 Hartford Express connects with PVTA Route #65 to downtown Springfield, from where you can connect to other PVTA buses to other parts of the Pioneer Valley. If you check the websites of PVTA and CT Transit, you can get more info on schedules and fares. This site has a rideshare offer for a female nonsmoking rider from Forest Park, Springfield, to Downtown Hartford. Marco polo (talk) 20:25, 5 December 2008 (UTC)[reply]

Miami tribe

Did the miami tribe of Indiana ever live close to the Great Lakes? —Preceding unsigned comment added by 169.204.229.142 (talk) 19:03, 5 December 2008 (UTC)[reply]

Strangely enough, there's an article on that: Miami tribe. Cheers. --Jayron32.talk.contribs 19:08, 5 December 2008 (UTC)[reply]
They lived on the St. Joseph River by Lake Michigan, and along the Maumee River ("Maumee" is a variation of "Miami") near Lake Erie. —Kevin Myers 23:20, 5 December 2008 (UTC)[reply]

Mention of Bao Zheng in the Water Margin

Someone told me the Water Margin mentions Judge Bao as being the reincarnation of the Wen Qu (Scholar or intellect star). Does anyone know what chapter this appears in the foreign press English translation? --Ghostexorcist (talk) 20:33, 5 December 2008 (UTC)[reply]

Eh? I was unaware that Bao Zheng was in Water Margin at all. Having read the foreign press translation, I don't think he was ever mentioned. bibliomaniac15 03:41, 6 December 2008 (UTC)[reply]

Customs and Body Language

I forgot what channel but I watched a clip of Yasser Arafat, President Bush and someone offical from Israel. They were all having a photo op outside some building and then Bush tried to escort them both into the building. Bush wanted to be the last to walk into the building. However Arafat and the Israel did not want to enter th building first. It almost looked like they were going to have a tussle because each tried to push the other first. Then the announcer said something about an middle eastern custom were the "alpha male" (sorry, don't have a better word) always enters the building last. What is this custom? Is there a list of other customs in Wiki? --Emyn ned (talk) 20:40, 5 December 2008 (UTC)[reply]

The "alpha male last" complex comes close but is a bit simplistic, I guess. Semitic custom requires anyone offered something to deny it at first. Say I offer you some chocolate cake, you are expected to say, "No, I couldn't," even though you really want it. Then the offerer is supposed to press you harder to accept. So you can see what could happen...
Bush: After you
Arafat: Oh no, you first, I insist.
Israeli Official: Ah, no, if you would do the honor...
etc.
Typically, I would imagine whoever is host of the meeting would go last. Wrad (talk) 22:57, 5 December 2008 (UTC)[reply]
It's not just a Semitic custom, it's considered polite (although perhaps not required to the same extent) in the UK, at least. --Tango (talk) 23:04, 5 December 2008 (UTC)[reply]
Oh, certainly there are varying shades of it in other cultures. Wrad (talk) 23:29, 5 December 2008 (UTC)[reply]
Alphonse and Gaston... AnonMoos (talk) 05:22, 6 December 2008 (UTC)[reply]
Here's a 2000 photo and caption from the Camp David Summit: heads of state Bill Clinton, Yasser Arafat, and Ehud Barak doing the "after you" routine. I can recall its being broadcast and widely perceived (as I recall in Israel) as goodnatured and revealing no actual reluctance to proceed. As regards that "alpha male" remark by Emyn ned's unidentified newscaster: it perhaps could be understood as an artifact reflecting the editorial policy of that particular media outlet, the level of professionalism of its staff, and its collective assessment of its material and audience. -- Deborahjay (talk) 15:49, 6 December 2008 (UTC)[reply]

Translation to Latin, please

Hi - I'm writing a scene where a character in renaissance times is performing a facial exercise called 'swallowing beauty'; can anyone translate that phrase to Latin for me please?

Thanks Adambrowne666 (talk) 20:59, 5 December 2008 (UTC)[reply]

The translation will depend on what that phrase means. Does it mean "a swallowing beauty" (a beautiful man/woman in the act of swallowing)? In which case, is it a male or female beauty? Or does it mean "the act of swallowing beauty", or swallowing in a way that removes beauty? Marco polo (talk) 21:10, 5 December 2008 (UTC)[reply]
Ah, good to see you, Ser Marco. Yeah, this is why using one of those online English/Latin dictionaries is no good, I suppose. The phrase means 'the act of swallowing beauty'. If possible, I'd prefer it if the translation was such that a lay person could work out what it meant. Adambrowne666 (talk) 21:16, 5 December 2008 (UTC)[reply]
Is swallowing beauty your own phrase, Adam? I ask because both words there are less than straightforward, and you may be able to say what you're after. I think Marco is asking, Is this about someone metaphorically gobbling up beautiful things and, if so, is it about possessing beautiful things and keeping them away from others or about feeding on them, so that they are gone? Xn4 (talk) 03:19, 6 December 2008 (UTC)[reply]
And of course, do you want a literal translation, or something idiomatically Latin that has a similar meaning (if we could find such a thing)? Adam Bishop (talk) 03:58, 6 December 2008 (UTC)[reply]
Thanks, guys, for all this. Think of it as a yoga pose, like Salute to the Sun, or Panther Startled by Vacuum Cleaner, or whatever - he's not actually swallowing anything, but performing an exercise of the muscles of the jaw and throat to build them up, with the aim of making himself more beautiful. So the suggestion might be that he is swallowing the beauty of the world into himself, and making himself more beautiful thereby...? Does that help? Adam, I imagine an idiomatic translation would be better. Adambrowne666 (talk) 06:04, 6 December 2008 (UTC)[reply]
Sorbere formam? "To swallow beauty" Very literal. Wrad (talk) 06:11, 6 December 2008 (UTC)[reply]
I think "haustus" would be better here, plus the genitive? So "haustus formae". Adam Bishop (talk) 07:18, 6 December 2008 (UTC)[reply]
Those suggestions certainly convey the notion of taking beauty into oneself, but I can't help thinking that something involving mandere (or manducare) or vorare might better fit the muscular movements such an exercise might involve. As for "beauty," why not pulchritudo as something that might be suggestive to a "lay person" (one who's encountered the word pulchritude, at least)? Mandere pulchritudinem? Deor (talk) 13:27, 6 December 2008 (UTC)[reply]
How about "absorbere"? That would be easily recognizable as "absorb". Adam Bishop (talk) 17:41, 6 December 2008 (UTC)[reply]

Where was this quote originally from?

"If you're so smart, why ain't you rich"? I have heard it as a common jab at people. What is its origin? And who first said it in what context?

128.100.123.154 (talk) 21:32, 5 December 2008 (UTC)[reply]

I can't answer for this particular expression, but the general notion of "If you're so X, why don't you do Y" goes way back, at least as far back as the time of Jesus. When he spent 40 days in the wilderness, the Devil tempted him with "If thou art the Son of God, command this stone that it become bread". And Jesus answered unto him, "It is written, Man shall not live by bread alone". Later, at his crucifixion, the rulers scoffed at him, "He saved others; let him save himself, if this is the Christ of God". And the soldiers mocked him, "If thou art the King of the Jews, save thyself". -- JackofOz (talk) 22:42, 5 December 2008 (UTC)[reply]
Just curious! What was Jesus' reply to that? Jay (talk) 05:36, 6 December 2008 (UTC)[reply]
Well, he asked God to forgive the soldiers, since they didn't really understand who he really was, anyway. Wrad (talk) 05:53, 6 December 2008 (UTC)[reply]
Don't know if it is the original context, but Louis Jordan had a hit song (written by Walter Bishop) called "If You So Smart, How Come You Ain't Rich?" in 1951. It's probably where that particular phrasing was popularized. It's the earliest the phrase is mentioned in any form in US newspapers according to ProQuest. --98.217.8.46 (talk) 23:30, 5 December 2008 (UTC)[reply]
Partridge, E., Beale, P. (1992). A Dictionary of Catch Phrases. OCLC 26628502 guesses around 1920 for original use, but Aristotle's tale of Thales and his olive presses in Politics[41] probably shows that the sentiment is a bit older.—eric 06:00, 6 December 2008 (UTC)[reply]

Private Citizen Bailout?

Is there a process by which a private US citizen can apply for a portion of the federal bailout money? Not for their business - just for their self or their family. —Preceding unsigned comment added by Bikingshaun (talkcontribs) 22:00, 5 December 2008 (UTC)[reply]

No. --Tango (talk) 23:02, 5 December 2008 (UTC)[reply]
"Private citizen bailout" is called unemployment benefits and welfare. --98.217.8.46 (talk) 23:20, 5 December 2008 (UTC)[reply]
Not exactly. We pay for unemployment benefits on every check. The mortgage and auto industry didn't pay for their bailouts. 67.184.14.87 (talk) 09:23, 6 December 2008 (UTC)[reply]
Er, they paid taxes just like every other company. Nil Einne (talk) 12:14, 6 December 2008 (UTC)[reply]
No, they did not pay any taxes for the bailouts. Not a single penny. 67.184.14.87 (talk) 13:44, 6 December 2008 (UTC)[reply]
Er they paid general taxes. Perhaps things work differently in the US but as far as I'm aware in the vast majority of countries welfare is paid for by general taxation too. You don't pay a special tax for welfare, nor do you pay more if you have used or may use welfare nor do you get a rebate if you have never used welfare. You still get welfare if you've never paid a cent in tax. So I don't really get your point. (None of this answers whether the bailouts or welfare are justified of course, that's irrelevant) Nil Einne (talk) 13:46, 6 December 2008 (UTC)[reply]
Every year, the President submits an annual budget to Congress. All income derived from taxes has already been allocated. In fact, the US government has been operating at a deficit for a long time now. The national debt currently stands at $59.1 trillion or $516,348 per household. 67.184.14.87 (talk) 16:05, 6 December 2008 (UTC)[reply]
So is your complaint it was not part of the budget? If so what did you mean by "We pay for unemployment benefits on every check"? I should point out that emergency spending that comes in between budgets is, I'm pretty sure, not unheard of in most countries, including I presume the US (actually these can be considered an additional budget or mini-budget). If your trying to argue that welfare is paid for by taxes but the bailouts are not because the US has a deficit, I could just as well argue that welfare is not paid for by taxes but by the deficit. Indeed you could even argue that you should cut out welfare for the next X years and use that money (which can be tax money if you want) to fund the bailouts if you want. Ultimately the whole argument is going nowhere. If you want to attack the bailout, you should attack it for other reasons like effectiveness, fairness, etc etc (and do so elsewhere since the RD isn't the place for such a discussion), not the moot point of whether the money is coming from taxation and whether thats the same source as welfare. (There's nothing wrong with opposing an increase in spending, but that's a different issue too and also not one for the RD) Ultimately what it comes down to is that the US government pays for all their spending including bailouts and welfare from the federal budget. Incoming for that budget comes form a variety of sources including taxation and debt. Trying to ringfence parts of the spending and say this comes from taxation, this comes from the debt doesn't make much sense unless perhaps the money is actually ringfenced when it is collected. Nil Einne (talk) 17:22, 6 December 2008 (UTC)[reply]
I'm not complaining about anything. I'm simply pointing out the fact that they did not pay taxes for the bailout. 67.184.14.87 (talk) 17:26, 6 December 2008 (UTC)[reply]
You don't pay taxes "for" something, you just pay taxes. They did pay taxes. --Tango (talk) 17:31, 6 December 2008 (UTC)[reply]
(Outdent) Sure you do. Unemployment benefits are paid through a tax on your income. Look up the Federal Unemployment Tax Act. But as I already pointed out, the budget for 2008 has already been passed and all the money allocated. The only way the auto industry could have possibly paid taxes for the bailout is if for some reason they were paying extra taxes. 67.184.14.87 (talk) 19:35, 6 December 2008 (UTC)[reply]
Yes that what I've been (trying to) saying all along. Nil Einne (talk) 17:35, 6 December 2008 (UTC)[reply]
Taxes have lots of different names, but that's all just political. Tax is tax. It all goes into one pot and it all comes out of one pot. The corporations paid corporation tax, that went into the public finances and the bailout came out of those same public finances. --Tango (talk) 20:03, 6 December 2008 (UTC)[reply]

December 6

University and College

Do a lot of people go to collge after university?99.226.138.202 (talk) 02:56, 6 December 2008 (UTC)[reply]

I think you need to identify which country you're asking about, 99.226.138.202. In the UK, for instance, the simple answer is No. Many people here go to both a college and a university at once, as Oxford, Cambridge, Durham and London are collegiate universities, but apart from that most institutions called 'colleges' in the UK are actually secondary schools or further education colleges and thus generally come before universities. Xn4 (talk) 03:33, 6 December 2008 (UTC)[reply]
In the U.S., it's common to refer to any four year undergraduate institution as a college: "Where are you going to college?" "Michigan" (meaning, the University of). And many schools with extensive postgrad programs (supposedly a hallmark of a university) call themselves colleges, like Boston College and the College of William and Mary. You'll hear "Irene's in college" or "Curt's in grad school," but you'll go quite some time before you hear "Leonard's at university" (except with a specific -- "He's at the University of Maine." There are also two year community colleges (sometimes called junior colleges), though I haven't heard of one of them calling itself a university. --- OtherDave (talk) 04:51, 6 December 2008 (UTC)[reply]
In Canada, where "university" and "college" are fairly distinct, it sometimes happens that a university graduate goes to college afterwards. Their university degree is probably abstract and impractical for the real world, so they go to college to get useful training for a career. Adam Bishop (talk) 06:27, 6 December 2008 (UTC)[reply]
With due respect to the poster above, in the UK many university graduates enrol in an FE college for a short course or part-time course to gain extra skills, for example digital image processing or British sign language. Itsmejudith (talk) 16:28, 6 December 2008 (UTC)[reply]

Bangladeshi districts history

I heard that Pabna and Sirajganj district were together as Pabna. So, what about other districts like Rangpur? I also heard that Gaibandha, Kurigram, Dinajpur and Lalmonirhat were together with Rangpur as Rangpur. —Preceding unsigned comment added by 74.14.116.254 (talk) 03:09, 6 December 2008 (UTC)[reply]

Unfortunately our articles on these Bangladeshi districts need much improvement. Itsmejudith (talk) 16:33, 6 December 2008 (UTC)[reply]

Winston Churchill

Is Winston Churchill an atheist? Sayaialahbejin (talk) 13:06, 6 December 2008 (UTC)Evan[reply]

Winston Churchill is dead so he isn't anything. Anyway I highly recommend you read the Winston Churchill article if you haven't already (and you should always check out an article before asking a question as you'll often get the answer a lot faster that way). In particular, the section on India makes it sound like he was an atheist although it doesn't say it explicitly. Saya ialah Nil Einne (talk) 14:01, 6 December 2008 (UTC)[reply]

Or at any rate, if he's dead and he DOES nevertheless have an opinion one way or the other, it's pretty safe to say, whatever he was in life, he's not an atheist now! (Sorry, couldn't resist.) —Preceding unsigned comment added by 71.104.139.75 (talk) 19:54, 6 December 2008 (UTC)[reply]

[42] suggests he was either agnostic or ambigious Nil Einne (talk) 14:39, 6 December 2008 (UTC)[reply]
Actually, this is an excellent question. The Wikipedia article on Churchill claims that he Angelican (in the Info Box) and another part claims he was an atheist. There's a small dicussion on this on the discussion page. I'm not sure if this issue was resolved. 67.184.14.87 (talk) 16:34, 6 December 2008 (UTC)[reply]
  • I know you just mistyped "Anglican" there, but I must say that "Angelican" has a delightful sound to it! --Anonymous, 18:13 UTC, December 6, 2006.
Thanks for that. Despite my suggestion to the OP, I didn't actually read the whole article;:-P and primarily searched for Christ, agnos & athei but somehow forgot religion and didn't think of anglican so missed that part Nil Einne (talk) 17:11, 6 December 2008 (UTC)[reply]
Not necessarily. It's perfectly possible there is life after death but there are no gods. Algebraist 19:55, 6 December 2008 (UTC)[reply]

Is spying illegal?

(Not a question about spying by the government) I'm asking about spying by one individual on another. —Preceding unsigned comment added by 199.76.172.209 (talk) 13:59, 6 December 2008 (UTC)[reply]

As the header says, we don't give legal advice. And as with nearly every question which asks, it it illegal, the first basic isssue answer is, it depends where you live. Add as is often also the case, it depends what you mean by spying. More specifically for examples, in many countries listening to or recording a private phone call without the permission of at least one party is illegal. Breaking into someone's house to install a camera or whatever will obviously be illegal. (If you are lawfully invited into the house, it may or may not be illegal I suspect depending on the circumstances) Recording or filming on private property where someone has a resonable expectation of privacy, without the owners permission will often also be illegal. Accessing someone's computer or email account or whatever without their permission will also often be illegal. Following someone too much could be considered stalking and in any case, they may also be able to take a restraining order out against you which will stop you from coming within a certain distance of them. In some situations, the police may decline to persue the matter or perhaps there will even be no criminal violation but you may still be able to take civil action against the violator. If you have a question about a specific situation then you should contact a lawyer who can advise you to precise legal situation in whatever jurisdiction concerns you. Nil Einne (talk) 14:20, 6 December 2008 (UTC)[reply]
There are many criminal laws in different jurisdictions about acts such as breaking and entering, trespass, accessing someone's computer without permission, recording telephone calls, stalking, etc. However in more cases, it is likely to be a civil matter e.g. nuisance, breach of confidence, civil trespass, trespass to chattels, breach of privacy/invasion of privacy (see e.g. Privacy laws of the United States), laws about trade secrets, etc. I suggest you read some of those articles unless you have a more specific question. --Maltelauridsbrigge (talk) 16:27, 6 December 2008 (UTC)[reply]

Much of it will be a matter of motive and degree. Technically, willful eavesdropping is a kind of spying, but a person probably won't be prosecuted for it. Setting up a hidden camera in someone else's private quarters is entirely different. And actions taken by a private citizen to uncover and prove criminal activity will be treated differently than those same actions would if they were done as part of a criminal enterprise (recording a conversation without the person's consent to take it to the police as opposed to doing so with the intent of using it to blackmail someone, for example).

Number of trees in the United states

Is it true that the number of trees in the United states is getting bigger each year? And what about Europe? Gridge (talk) 14:21, 6 December 2008 (UTC).[reply]

You can get pretty graphs for the U.S. here. This is the main pertinent quote: "Since 1900, forest area in the U.S. has remained statistically within 745 million acres +/-5% with the lowest point in 1920 of 735 million acres. U.S. forest area in 2000 was about 749 million acres." -- kainaw 18:36, 6 December 2008 (UTC)[reply]

(Heralrdy) Royal coat of arms of the UK and predecessor kingdoms

Which supporters would these coat of arms have had before being merged into the coa of the UK? Links to images would be very welcome! Thanks! ;) --217.227.109.160 (talk) 18:20, 6 December 2008 (UTC)[reply]

England -- Lions (in Tudor times, a lion and a dragon).
Scotland -- Unicorns. AnonMoos (talk) 18:55, 6 December 2008 (UTC)[reply]

Source of quotation

where can I find the original source of the quote, "the more I know, the more I don't know."  ? I need it for a paper in my psychology class. Thank you —Preceding unsigned comment added by Ba2011 (talkcontribs) 19:37, 6 December 2008 (UTC)[reply]

Scholarly edition of Grimm's Fairy Tales

I'm very interested in folklore studies, perhaps even in pursuing it as a career. I would like to obtain a scholarly edition of the work of Jakob and Wilhelm Grimm - not a cute illustrated children's edition, or one meant for children at all, but one with the stories exactly as they were collected (unpleasant details intact, violence and incest and all), and preferably annotations and footnotes meant to enhance the tales for those who would study them for cultural, literary, and psychological significance, not simply read them for entertainment. Unfortunately, I haven't yet studied German, so this would have to be an English edition (though I wouldn't object to a bilingual English/German edition so long as it was unabridged and the scholarly folkloristic elements, in English, were present). I know there may not be a "definitive" edition accepted by all scholars, but can anyone make a solid recommendation? - Aletheia —Preceding unsigned comment added by 71.104.139.75 (talk) 19:45, 6 December 2008 (UTC)[reply]

  1. ^ Hubbard, The Creation of Human Ability.